SlideShare a Scribd company logo
1 of 33
Download to read offline
1
LICENSURE EXAMINATIONS FOR TEACHERS (LET)
Reviewer for ENGLISH
Competencies:
 Arrive at meanings of words using context clues, structural analysis and other word formation
techniques
 Note details to discover the central theme of a passage
 Point out the organization structure of a passage and determine how the parts are related to the
whole
 Draw inferences and implications on reading texts
 Distinguish the different parts of speech in English – their meanings, forms, order, and functions
 Analyze sentences in terms of their constituents elements
 Identify the rules of grammatical usage
Prepared by:
Ruth A. Alido
Philippine Normal University, Manila
VOCABULARY
Vocabulary knowledge implies a rich understanding of the word. It means knowing a word by definition
and associating experiences with that word.
1. Structural Analysis. Words are made up of the smallest meaningful units called morphemes. The
visual scrutiny of unfamiliar words to identify morphemes is called structural analysis. Knowing the
roots, prefixes, and suffixes of words helps reveal the meaning of the total word form.
a) Root words are words from which other words are formed by adding a beginning part (prefix) or
an ending part (suffix)
active (move) porter (carry) contradiction (to speak)
b) Prefixes are word parts added to the beginning of a word
benevolent (good) decline (from) nonsense (not)
c) Suffixes are syllables added at the end of a word to form a new word with a different meaning
Biology (study of) homeless (without) scientist (one who does)
2. Word Formation. Words undergo changes. The following are five processes of word formation:
a) Clipping means to cut off the beginning or the end of the word. It may mean cutting from both
ends, leaving a part to stand for the whole.
phone photo psycho trigo chem.
b) Blending is formed by fusing or putting two words together. Usually the first part of one word
is blended or fused with the last part of another. The blended word then gets its meaning from
the two words put together.
Eurasian Philhealth cosmonaut smog telecast
c) Compounding uses two or three words put together to make a full form. Most often the
meaning of the word is different from its parts. Sometimes it is the meaning of the two words
put together.
tightwad blackout first aid runner-up trigger-happy
d) Acronymy is the use of initial letter or syllables of several words in succession.
PPSTA UNESCO AWOL scuba radar
e) Folk or Popular Etymology results from changing a word in part or in whole to make it more
like a familiar word.
belfry isle bachelor barbeque caesarian
2
3. Context Clues. The meaning of a word may be determined by its environment – the words that
surround it, either coming before or after it in the sentence of in the paragraph.
a) Definition is considered the simplest and most obvious way by which the meaning of a word is
revealed. The be verb is used to equate the term to be defined to the familiar word in the
sentence.
Psychiatry is the branch of medicine that deals with the diagnosis, treatment, and
prevention of mental disorders.
b) Restatement may be in the form of synonyms, examples, elaboration by the use of modifiers,
and pairing closely related words. It is usually introduced by signal words: that is, for example,
such as, like, in the way, that, in other words, what this means, etc. It may also be signaled by
the dash and the parentheses.
He was a true peripatetic, like the walking philosophers who followed Aristotle and the
wandering Jews of Israel.
c) Synonym is a word that means essentially the same thing as another word. It is usually preceded
by the function word or. Sometimes the synonym may be found in another sentence within the
paragraph.
God is omniscient, or all-knowing.
d) Antonym is the opposite or contrasting word that may serve as a clue to the unfamiliar word.
Julia’s remarks are generally not nebulous, but clear.
e) Inference requires reading between lines to see connections and relationships not explicitly
stated in a particular passage.
The practicing physician as a rule is completely unskilled in obstetrics. His preliminary
training contains little or nothing concerning the details of pregnancy and childbirth.
f) Homophones & Words with Multiple Meanings indicate several meanings that a word has in
English.
Susan bought a bar of soap.
The soldiers tried to bar the enemies.
The brilliant student passed the bar examination.
g) Words of Various Disciplines are the basic meaning of specialized vocabulary in a particular
subject area.
The capital of the Philippines is Metro Manila. (Social Science)
His capital for the new business is one million pesos. (Math)
h) Multiple Context make the meaning of unfamiliar words clearer and easily understood.
Butterflies fly from flower to flower.
How high did the boy fly his kite?
Airplanes fly regularly from Manila to Hongkong.
4. Idioms. An idiom is an expression peculiar to a language. It usually has several associated meanings
which are not readily understandable from its grammatical construction and cannot be derived from
the meaning of its separate elements.
 cross to bear – endure a kind of burden
 close-fisted – a miser
 cold-reception – unfriendly
 broad daylight – open, full daylight
 blackmail – money extorted by threat of intimidation
 Argus-eyed – jealously watchful
 what not -etcetera
 blue-blood – noble blood
 chicken-hearted – a timid, cowardly fellow
 absent-minded – to be inattentive of what is going on
 fair weather friend – a friend who deserts you in time of difficulties
 henpecked husband – a submissive husband
 laughing stock – object of ridicule
 greenhorn – an inexperienced person
 by heart – from memory
 burn the midnight oil – study or work far into the night
 bundle from heaven – new-born baby
 belle of the ball – most popular girl
 blanket authority – complete right or privilege
 behind the times – unprogressive
 came to terms – deal with misunderstanding
3
5. Figures of Speech. These forms of expressions are used to convey meaning or heighten effect, often
by comparing or identifying one thing with another that has meaning or connotation familiar to the
reader or listener.
a) Simile – a comparison between two unlike objects by using like or as
His mind is like a sponge.
b) Metaphor – an indirect comparison of unlike objects
She is a phantom of delight
c) Personification – the giving of human characteristics & capabilities to nonhuman things such as
inanimate objects, abstract ideas, or animals
The clouds cried a torrent of tears.
d) Apostrophe – an address to the absent as if present or to the inanimate as if human
“O wind, if winter comes, can spring be far behind?”
e) Hyperbole – the use of excessive exaggeration for effect
Waves mountain high broke over the reef.
f) Oxymoron – the combining of contraries to portray a particular image or to produce a striking
effect
Parting is such sweet sorrow.
g) Paradox – uses a phrase or statement that on surface seems contradictory, but makes some kind
of emotional sense
Let us go to war for peace.
h) Metonymy – substitutes a word that closely relates to a person or thing
Have you no respect for gray hairs?
i) Synecdoche – uses a part to represent the whole
No busy hand provoke a tear/No roving foot shall crush thee here
j) Litotes – makes a deliberate understatement used to affirm by negating its opposite
Regine Velasquez is not a bad singer.
k) Irony – the opposite of what is expected is what happens
To cry like a baby, that’s a fine way for a man to act.
l) Allusion – refers to a literary, biblical, historical, mythological, scientific event, character, or
place
Beware of the kiss of Judas!
m) Antithesis – a contrast of words or ideas
She looks like an innocent flower but watch out for the serpent under it
6. Rhetorical Devices. These are sound devices used to convey meaning through rhyme and rhythm.
a) Onomatopoeia – uses a word having a sound that imitates what it denotes
hiss, bang, buzz, hush , swoosh
b) Alliteration – involves the repetition of initial consonant sounds
wicked and wan, threatening throngs
c) Assonance – uses repetition of vowels without repetition of consonants, also called a vowel
rhyme
alone, alone, all, all, alone
d) Consonance – repeats the final consonant sounds, also called a slant rhyme
dreary and weary; odds and ends
e) Rhyme – employs identical sounds from the vowel of the accented syllables to the end
hold, told, mold, gold; die, sky, my, fly, pie
f) Anaphora – repeats a word or words at the beginning of two or more successive clauses or
verses
Cannons to the right of them/Cannons to the left of them
V OCABULARY PRACTICE
A. Give the meaning of the underlined word based on the meaning of its root given in the parenthesis
after each sentence.
1. A portable (to carry) video CD is very convenient. _________________________.
2. The people admire their leader for his tenacious (to hold) stand. ______________________.
3. The testimony of the witness was credible (to believe). _________________________.
4. The old man’s faith guided him in the tortuous (to twist) path of life. __________________.
5. The old theater had to be renovated (new). _________________________.
6. His unpleasant remarks caused a rupture (to break) in their friendship. _________________.
7. The spectators (to see) were amused by the magician’s clever antics. ___________________.
4
8. The aid that the victim received from civic spirited citizens was superfluous (to flow). _____.
9. The crowd became bored listening to the loquacious (to speak) politicians. ______________.
10. Syphilis is a malignant (evil) disease. _________________________.
B. Pick out the meaning of the underlined word in the sentence following the meaning of the prefix.
1. A bilingual (two) dictionary is very helpful to students of philology.
A. many languages C. a single language
B. two languages D. the second language
2. After many years of foreign rule, the Philippines enjoyed autonomy (self).
A. self-government C. freedom of speech
B. peace and prosperity D. self-expression
3. The body was exhumed (out of) for another autopsy.
A. buried B. consecrated C. dug out D. hung out
4. Let this fine gesture be the precedence (before).
A. preparation B. priority in time C. example of prudence D. presentation
5. The panorama (whole) in Batanes is breathtakingly beautiful.
A. entire view B. live music C. landscape D. natural resource
6. King George abdicated (away from) in favor of his brother.
A. objected to his brother C. denounced his throne
B. renounced his throne D. abducted his brother
7. The comedian used many hyperboles (over) to make his audience laugh.
A. allusions B. comparisons C. exaggerations D. contrasts
8. Patients suffering tuberculosis were segregated (apart).
A. placed with others C. refused admission
B. placed in pay wards D. separated from others
9. Worry and fatigue can befog (completely) our thinking.
A. obscure B. clarify C. becalm D. bedeck
10. A person with that personality cannot remain a non-entity (not) for long.
A. one who lacks money C. one of no importance
B. one of no power D. one without identity
C. Choose the correct suffix for the word that best fit the blank.
1. The physician was conducting a __________ research.
A. scientism B. scientific C. scientist
2. The liquor is strong. It has high __________.
A. potential B. potentate C. potency
3. The body cannot be identified. It is at the state of __________.
A. decomposition B. decomposable C. decomposer
4. Mr. Flavier worked in favor of population control. He is an __________ of slavery.
A. advocacy B. advocate C. advocation
5. After a year’s stay abroad, the scholar was glad to turn __________.
A. homely B. homeless C. homeward
6. The general manager gave Ms. Santos a duty to perform. Ms. Santos was the __________.
A. employer B. employee C. employment
7. The criminal was very sorry for the crime he committed. He was __________.
A. repenter B. repentant C. repentance
8. The doctor announced that his sickness was not serious it was __________.
A. remedy B. remedial C. remediable
9. The professor used terms that were too general. His students wished he would be more ____.
A. specific B. specify C. specification
10. The senator is engaged in the production, distribution, and consumption of wealth and of the
means of supplying the material needs of his constituents. He is a well-known __________.
A. economist B. economics C. economy
D. Choose the correct meaning of the underlined idiom in the sentence.
1. The wife shed crocodile tears when her cruel mother-in-law died.
A. sincere grief B. pretended sorrow C. continuous flow of tears
2. When the foreigner first stepped on the Philippine soil, he was all at sea about everything.
A. confused B. at fault C. disappointed
5
3. The newly married couple executed a French leave before the reception was over.
A. an aristocratic way of saying good-bye
B. leaving without giving due notice of one’s departure
C. waving one’s hands during departure
4. The daughter bares her soul to her mother.
A. confides with some reservation
B. opens for inspection
C. confesses innermost thoughts
5. From the sixth floor of the building, we got a bird’s eye view of the city.
A. view from a telescope
B. view over the entire area
C. photograph of a bird
6. They are trying to find out what is really at the bottom of the continuous rise in the prices of
commodities.
A. purpose B. reason C. procedure employed
7. His parent’s hopes turned to ashes in the mouth when he married soon after graduation.
A. overwhelming joy B. burned to ashes C. bitter disappointment
8. The enemies entered the city armed to the teeth.
A. heavily armed B. with covered mouths C. with weapons between the teeth
9. The audience was all ears to the distinguished orator.
A. covering both ears
B. making a compliment
C. listening very attentively
10. The man is very angry. Don’t speak a single word otherwise you will be adding fuel to the fire.
A. provoking further strife
B. making his illness worse
C. adding firewood to make the fire alive
E. Identify the figures of speech and rhetorical devices used in the sentences below.
1. The dragonfly huge like a blue thread loomed from the sky.
2. Sometimes too hot the eye of heaven shines.
3. She sells seashells on the seashore.
4. Each flower has wept and bowed toward the east.
5. Ten thousand daffodils saw I at a glance.
6. Mighty Hades, why, they power hovers!
7. I did not realize I had a Hamlet for a husband.
8. There is emptiness in wholeness.
9. Her speech is a newly sharpened scythe.
10. Address that question to the chair.
11. The poor need roofs over their heads.
12. The night has a thousand eyes.
13. Oh, Liberty! How many crimes are committed in thy name!
14. The United Nations Forum is a tower of Babel.
15. Sad storms whose tears are vain.
READING COMPREHENSION
Reading comprehension questions usually fall into several general categories.
1. Main Idea. This usually refers to the passage as a whole, not to some segment or part of the
passage. Questions are usually about the main idea or theme of the passage, about a possible title,
or about the author’s primary objective. The main idea is typically (but not always) found in the
first paragraph. It is the statement that gives the overall theme of the passage. In many cases, it
is in the form of an argument, including a premise and conclusion.
6
World War II brought new demands and needs to the nation in the 1940s.
Financing the war meant additional taxes and changes to payroll processing. An armed
force with millions of people produced new challenges in administration and record
keeping. New weapons required countless calculations and tests. Defense research
demanded the development of large-scale computing devices.
2. Supporting Ideas. This is about the idea expressed in one part of the passage rather than about
the passage as a whole. This type of question is to distinguish between the main idea and those
themes that support it, some of which may be implicit or implied rather than explicitly stated.
Males and females are each associated with different kinds of behavior, and what
is considered masculine and feminine differs from society to society. These concepts of
masculinity and femininity extend to how people walk, sit, talk, and dress. In our society,
as in all others, men walk and talk in certain ways and until very recently dressed very
differently from women. In some societies, different spatial areas are associated with
males and females – women in many Middle Eastern societies are restricted to certain
parts of the house and may only come into contact with the males who are members of
their family. In such societies, the coffee house and the market are defined as male
domains. In contrast, in some African societies, women predominate in the marketplace.
3. Drawing Inferences. This asks about ideas that are not explicitly stated in a passage. The
question refers meanings implied by the author based on information given in the passage.
The procedure is actually quite simple. First, you arrange the items into different
groups. Of course one pile may be sufficient depending on how much there is to do. If
you have to go somewhere else due to lack of facilities that is the next step; otherwise,
you are pretty well set. It is important not to overdo things. That is, it is better to do too
few things at once than too many. In the short run this may not seem important but
complications can easily arise. A mistake can be expensive as well. At first, the whole
procedure will seem complicated. Soon, however, it will become just another facet of
life. It is difficult to foresee any end to the necessity for this task in the immediate future,
but then, one never can tell. After the procedure is completed one arranges the materials
into different groups again. Then they can be put into their appropriate places.
Eventually they will be used once more and the whole cycle will then have to be
repeated. However, that is part of life.
4. Specific Details. This type of questions asks about specific facts or details the author has stated
explicitly in the passage.
Today, farmers grow about 449 million tons of corn worldwide. In terms of area
under cultivation, it’s the world’s second or third largest crop. Providing 19 percent of
the world’s food calories and 15 percent of its food-crop protein, global corn production
yields about 200 pounds of the cereal for every individual alive. It should come as no
surprise, then, that corn – also known as maize – is a staple for some 200 million people,
including nearly half the world’s chronically malnourished. However, conventional corn
is deficient in the vitamin niacin, and roughly half its protein in lacks lysine and
tryptophan – two essential amino acids. As a result, this staple is not a sufficient protein
source, for children, the sick, and pregnant or lactating women.
5. Tone or Attitude of the Passage. This concentrates on the author’s style, attitude, or mood. The
use of key words such as adjectives that reveal if the author is “pessimistic,” “critical,”
“supportive,” or “objective” about an event, idea, or situation in the passage help determine the
tone or attitude.
Them dirty lousy politicians is getting altogether too high and might, the way
they is always arranging to take advantage of the little businessmen by raising up the tax
payments and collecting more money from the little fellows. They ain’t nothing much
can be done about this here business, because them politicians has certainly got the inside
connections and they always work through undercover arrangements. It’s might funny
that the generals and the admirals and the presidents of the big corporations aren’t paying
out no oversized tax installments but just the little businessmen who aren’t getting much
money nohow.
7
6. The Logical Structure of the Passage. This tests the overall meaning, logic, or organization of
a passage. The question asks how several ideas in a passage are interrelated or how a passage is
constructed, classifies, compares, describes events, or situations.
Let’s begin then to investigate the characteristics of your memory system as it
now operates. You may be surprised to discover that there is already more to your
memory system than you realize even existed. Scientific investigations of memory and
how it works have turned up the fact that each of us actually has three completely
different types of memory. These memory systems are called immediate memory, short-
term memory, and long-term memory. Each of these retains, and loses, information
differently. In addition, the life span of information within each system varies.
Consequently, each is use for different purposes and we must learn how to use each most
effectively. Whenever something is to be remembered for only a short period, it can go
into short-term memory; it if is to be used immediately, immediate memory is where it
belongs.
7. Determining the Meaning of Words from the Context. When a question asks for the meaning
of a word, it can usually e deduced from the context of the passage.
Nature loves edges. Take a place where mountain meets plain, field joins forest,
or river fuses with ocean. Abundant wildness usually occurs where one kind of habitat
hits another. At a good edge the number and variety of living things – the quotient of
ecological diversity – generally exceed what can be found in a more uniform habitat.
And because humans appreciate visual contrasts, edges often mean aesthetic distinctness
too. So it is with an estuary, the biological hot spot and scenic climax where freshwater
and saltwater first meet head on.
PRACTICE on BASIC READING SKILLS
1. Skimming and Scanning. Skimming is the selective reading of material to survey the purpose
and the general idea. Scanning, on the other hand, is a quick search for a specific information in
the text.
Read the selection below and answer the questions that follow. The text is written in such a
way to practice skimming and scanning.
Today’s English has many words…early myths. Some…changed…not
recognizable…mythological. Others remain…original form.
Greeks believe…beginning…things…existed…disorder…empty space…
condition…space…called chaos…today means disorder…complete disorganization…
described…chaotic.
…chaos…conceived…ordered world called ge…origin…source…things. Today
ge…common part…English words…related…earth, i.e., geology, geography, geometry.
…Greek god…sky was Uranus. Today we have…planet…Uranus…chemical element
uranium…rain…came…sky fertilized earth…union came… Titans,… race… giants…great
strength…anything…great size…strength…called titanic. One…titans was Saturn. One of the
planets…called Saturn…day…Saturday named…him…word
saturnine describes…gloomy person.
…Titans,…new generation…gods came…father was Jove…adjective, jovial means
‘cheerful.’ Jove lived…heavens…brother Hades, god…dead, lived below… sister. Ceres,
was…goddess…agriculture…cereal…named…her.
1. The word chaotic
A. means ‘erotic’ C. means ‘something disorganized’
B. is the same as chaos D. is a myth
2. The Greeks thought of the word ge which means
A. earth C. origin of all things
B. heaven D. rain
3. The Titan god of the sky was
A. Uranus B. Ceres C. Jove D. Hades
8
4. The statement that best approximates the main idea is
A. English is a combination of many languages.
B. Many words from Greek mythology have undergone little change in meaning.
C. The myths of early culture have enriched language.
D. English contains many words from Greek mythology which have altered in meaning and
in form.
5. A good title for this selection is
A. The Mystery of the English Language
B. The Influence of the Past on Language
C. Classical Mythology in Everyday English
D. The Varied Meanings of Words.
2. Finding the Topic Sentence. The topic sentence gives the key to an entire paragraph. It is
usually found in the beginning of a paragraph. However, there is no absolute rule. Some
paragraphs build up to a conclusion hence the key sentence is at the end.
Identify the topic sentence in each of the following examples.
Example 1.
The world faces a serious problem of overpopulation. Right now many people starve
from lack of adequate food. Efforts are being made to increase the rate of food production, but
the number of people to be fed increases at a faster rate.
Topic Sentence: _______________________________________________________
Example 2.
During the later years of the American Revolution, the Articles of Confederation
government was formed. This government suffered severely from a lack of power. Each state
distrusted the others and gave little authority to the central or federal government. The Articles of
Confederation produced a government that could not raise money from taxes, prevent Indian
raids, or force the British out of the United States.
Topic Sentence: _______________________________________________________
Example 3.
They had fewer men available as soldiers. Less than one-third of the railroads and only a small
proportion of the nation’s industrial production was theirs. For most of the war their coastline
was blockaded by Northern ships. It is a tribute to Southern leadership and the courage of the
people that they were not defeated for four years.
Topic Sentence: _______________________________________________________
3. Finding the General Theme. A more advanced skill is the ability to read several paragraphs and
relate them to one general theme or main idea. This involves careful reading of the entire passage
and deciding which idea is the central or main one. The general theme is usually most frequent or
most important and every sentence relates to it. In order to find the general theme:
a. Read at normal speed.
b. Locate the topic sentence in each paragraph.
c. Note ideas that are frequent or emphasized.
d. Find the idea to which most of the passage is related.
Identify the general theme in the example below. Note the underlined parts as you read the
passage.
True democracy means direct rule by the people. A good example can be found in a
modern town meeting in many small New England towns. All citizens aged twenty-one or over
may vote. They not only vote for officials, but they also get together to vote on local laws (or
ordinances). The small size of the town and the limited number of voters make this possible.
In the cities, voters cast ballots for officials who get together to make the laws. Because
the voters do not make the laws directly, this system is called indirect democracy or
representative government. There is no problem of distance to travel, but it is difficult to run a
meeting the hundreds of thousands of citizens.
Representation of voters and direct voice in making laws are more of a problem in state
or national governments. The numbers of citizens and the distances to travel make representative
government the most practical way to make laws.
9
The main theme of the passage is that
A. the United States is not democratic
B. representative government does not make laws
C. every citizen makes laws directly
D. increasing populations lead to less direct democracy
4. Finding Logical Relationships. The best way to fully understand the meaning of a passage is to
look for the general theme and then relate the ideas and opinions found in the passage to this
general theme. It is important to determine not only what is important in the passage but also
how the ideas interrelate to form the whole.
As you read the passage, look for general theme and supporting facts, words or phrases that
signal emphasis or shift in thought, and the relation of one idea to another.
The candidate who wants to be elected pays close attention to statements and actions that
will make the voters see him favorably. In ancient Rome candidates wore pure white togas (the
Latin word candidates means “clothed in white”) to indicate that they were pure, clean, and
above any “dirty work.” However, it is interesting to note that such a toga was not worn after
election.
In more modern history, candidates have allied themselves with political parties. Once a
voter knows and favors the views of a certain political party, he may vote for anyone with that
party’s label. Nevertheless, divisions of opinion develop, so that today there is a wide range of
candidate views in any major party.
The best conclusion to be drawn from the first paragraph is that after an election
A. all candidates are dishonest
B. candidates are less concerned with symbols of integrity
C. candidates do not change their ideas
D. policies always change
5. Making Inferences. An inference is not stated. It is assumed by the reader from something said
by the writer. An inference is the likely or probable conclusion rather than the direct, logical one.
It usually involves an opinion or viewpoint that the writer wants the reader to follow or assume.
Look for clues that reveal the writer’s viewpoint in the passage below.
Example.
Slowly but surely the great passenger trains of the United States have been fading from the rails.
Short-run commuter trains still rattle in and out of the cities. Between major cities you can still
find a train, but he schedules are becoming less frequent. The Twentieth Century Limited, The
Broadway Limited, and other luxury trains that sang along the rails at 60 to 80 miles an hour are
no longer running. Passengers on other long runs complain of poor service, old equipment, and
costs in time and money. The long distance traveler today accepts the noise of jets, the
congestion at airports, and the traffic between airport and city. A more elegant and graceful way
is becoming only a memory.
1. With respect to the reduction of long-run passenger trains, this writer expresses
A. regret B. pleasure C. grief D. anger
2. The author seems to feel that air travel is
A. costly B. slow C. elegant D. uncomfortable
READING COMPREHENSION STRATEGIES
 Answer passages with familiar subject matter first.
 Read the questions first, then the passage.
 Underline parts of the passage.
 Read all the answer alternatives.
 Learn to identify the major question types.
10
GRAMMAR
It is important to recognize common errors in grammar and usage based on the basic rules of grammar.
A. Verb Errors
1. Verb Tense. Check if the correct verb tense has been used in the sentence.
 When I came home, the children still didn’t finish dinner.
 When I came home, the children still hadn’t finished dinner.
In reported speech, check that the rule of sequence of tenses has been observed.
 She promised she will come.
 She promised she would come.
2. Tense Formation. Know the past participle of irregular verbs.
 He throwed it out the window.
 He threw it out the window.
3. Subject-Verb Agreement. Check if the verb agrees with the subject in number.
 There is many reasons why I can’t help you.
 There are many reasons why I can’t help you.
4. Conditional Sentences. The word if will NEVER be followed by the words will or would.
 If I would have known, I wouldn’t have gone.
 If I had known, I wouldn’t have gone.
5. Expressions of Desire. Unfulfilled desires are expressed by the form “________ had hoped
that ________ would (or could, or might) do ________.”
 I wish I heard that story about him before I met him.
 I wish I had heard (or could have heard or would have heard) that story about him
before I met him.
6. Verbs Followed by Verb Words. A verb word is the infinitive without the to.
 She ignored the doctor’s recommendation that she stops smoking.
 She ignored the doctor’s recommendation that she stop smoking.
7. Tag Endings. Check for three things in tag endings: a) Does the ending use the same person
as the sentence verb? b) Does the ending use the same tense as the sentence verb? c) If the
sentence verb is positive, is the ending negative; if the sentence verb is negative, is the ending
positive?
 She’s been there before, isn’t she?
 She’s been there before, hasn’t she?
8. Negative Imperatives. There are two forms for negative imperatives.
 Would you please don’t smoke here.
 Please don’t smoke here. or Would you please not smoke here.
9. Affirmative and Negative Agreement of Verbs. There are two correct forms for both the
affirmative and negative agreements.
 I haven’t seen the film and hasn’t either.
 I haven’t seen the film and she hasn’t either.
or I haven’t seen the film and neither has she.
10. Infinitives of Gerunds in the Complement of Verbs. Some verbs may be followed by
either an infinitive or a gerund. Others may require either one or the other for idiomatic
reasons.
 I intend learning French next semester.
 I intend to learn French next semester.
11. Verbs Requiring How in the Complement. The verbs KNOW, TEACH, LEARN, and
SHOW require the word how before an infinitive in the complement.
 She knows to drive.  She knows how to drive.
11
12. Idiomatic Verbs Expressions. There are a few commonly used idiomatic verb expressions.
a. must have (done) – it is a logical conclusion
They’re late. They must have missed the bus.
b. had better (do) – it is advisable
It’s getting cold. You had better take your coat.
c. used to (do) – was in the habit of doing in the past
I used to smoke a pack of cigarettes a day, but I stopped.
d. to be used to – to be accustomed to
The noise doesn’t bother me; I’m used to studying with the radio on.
e. make someone do – force someone to do
My mother made me take my little sister with me to the movies.
f. would rather – would prefer
I would rather you didn’t speak to her.
B. Pronoun Errors
1. Pronoun Subject-Object. Check if a pronoun is the SUBJECT or the OBJECT of a verb or
preposition.
 All of us – Fred, Jane, Alice, and me – were late.
 All of us – Fred, Jane, Alice, and I – were late.
2. Who and Whom. When in doubt about the correctness of WHO/WHOM, try substituting
the subject/object of a simpler pronoun to clarify the meaning.
 I don’t know who Sarah meant.
 I don’t know whom Sarah meant.
3. Pronoun Subject- Verb Agreement. Check if the pronoun and its verb agree in number.
 Jessa is absent, but a few of the class is here.
 Jessa is absent, but a few of the class are here.
4. Possessive Pronoun Agreement. Check if possessive pronouns agree in person and number.
 If anyone calls, take their name.
 If anyone calls, take his name.
5. Pronouns After the Verb To Be. TO BE is an intransitive verb and will always be followed
by a subject pronoun.
 It must have been her at the door.
 It must have been she at the door.
6. Position of Relative Pronouns. A relative pronoun refers to the word preceding it. If the
meaning is unclear, the pronoun is in the wrong position.
 He could park right in front of the door, which was very convenient.
 His being allowed to park right in front of the door was very convenient.
7. Parallelism of Impersonal Pronouns. In forms using impersonal pronouns, use either
“one… one’s/his or her” or “you… your.”
 One should take your duties seriously.
 One should take one’s/his or her duties seriously.
 or You should take your duties seriously.
C. Adjective and Adverb Errors
1. Use of Adjectives and Adverbs. Check if a word modifier is an adjective or an adverb and
make sure the correct form is used.
 I sure wish I were rich!  I surely wish I were rich!
2. Adjectives with Verbs of Sense. Intransitive verbs are described by adjectives while
transitive verbs are modified with adverbs.
 She looked very well.  She looked very good!
12
3. Comparatives. In using adjectives of one or two syllables ending in –y, add –er. Other
words of more than one syllable use more. Adverbs of one syllable add –er; longer adverbs
use more.
 This exercise is harder then the last one
 This exercise is harder than the last one.
4. Parallel Comparisons. Check if the correct form is used in parallel comparisons.
 The more you practice, you will get better.
 The more you practice, the better you will get.
5. Illogical Comparatives. Check comparisons to make sure they make sense.
 Texas is bigger than any state in the United States.
 Texas is bigger than any other state in the United States.
6. Identical Comparisons. Something can be the same as or like something else. Do not mix
up the two forms.
 Your dress is the same like mine.
 Your dress is like mine. or  Your dress is the same as mine.
7. Idioms Using Comparative Structures. Some idiomatic terms are formed like
comparatives although they are not true comparisons.
 You may have to spend so much as two hours waiting.
 You may have to spend as much as two hours waiting.
8. Noun – Adjectives. When a noun is used as an adjective, treat it as an adjective. Do not
pluralize or add ‘s.
 You’re talking like a two-years-old child!
 You’re talking like a two-year-old child!
9. Ordinal and Cardinal Numbers. Ordinal numbers (first, second, third, etc.) are preceded
by the. Cardinal numbers (one, two, three, etc.) are not.
 We missed first act.  We missed the first act.
10. Modifying Countable and Non-countable Nouns. If a noun can be preceded by a number,
it is a countable noun; if not it is uncountable.
 I was surprised by the large amount of people who came.
 I was surprised by the large number of people who came.
D. Errors in Usage
1. Connectors. Do not mix different forms in connecting ideas.
 She speaks not only Spanish but French as well.
 She speaks Spanish and French.
 She speaks Spanish. She also speaks French.
 She speaks Spanish and French too.
 She speaks not only Spanish but also French.
 She speaks both Spanish and French.
 She speaks Spanish as well as French.
2. Question Word Connectors. When a question word such as when or what is used as a
connector, the clause that follows is not a question. Do not use the interrogative form.
 Do you know when does the movie start?  Do you know when the movie starts?
3. Because. It is incorrect to say: The reason is because… Use: The reason is that…
 The reason he was rejected was because he was too young.
 The reason he was rejected was that he was too young.
 He was rejected because of his young age.
 He was rejected because he was too young.
4. Purpose Connectors. The word so by itself means therefore. So that means in order to or in
order that.
 We took a cab so we would be on time.  We took a cab so that we would be on time
13
5. Dangling Modifiers. An introductory verbal modifier should be directly followed by the
noun or pronoun that it modifies. Such a modifier will star with a gerund of participial phrase
and be followed by a comma. Look for the modified noun or pronoun immediately after the
comma.
 Seeing that the hour was late, it was decided to postpone the committee vote.
 Seeing that the hour was late, the committee decided to postpone the vote.
6. Parallel Construction. In sentences containing a series of two or more items, check to see if
the same form has been used for all the items in the series. Do not mix infinitives with
gerunds, adjectives with participial phrases or verbs with nouns.
 The film was interesting, exciting, and it was made well.
 The film was interesting, exciting, and well-made.
7. Unnecessary Modifiers. In general, the more simply an idea is stated, the better it is.
 That depends on the state of the general condition of the situation.
 That depends on the situation.
8. Commonly Confused Words. Be aware of the commonly misused words in English.
 He was laying in bed all day yesterday.
 He was lying in bed all day yesterday.
9. Misused Words and Prepositional Idioms. Take note of the prepositions in common
idioms.
 They came despite of the rain.
 They came in spite of the rain or  They came despite the rain.
GRAMMAR PRACTICE
A. Mark the letter of the poorly constructed sentence in each group.
1. A. She was a fine tennis player and she did not win a gold medal.
B. The refugees had neither food nor shelter for ten days.
C. He blushes; therefore, he is guilty.
D. He declared that he would never give up; nevertheless, he gave up.
2. A. Having no vacation you do not think about what a person could do if he had one.
B. In writing a paragraph, we shall find it easier to stick to our subject if we begin with a
topic sentence.
C. One never know what one may do in a moment of excitement.
D. He didn’t want to argue with people, but he could not ignore unfair criticisms
3. A. Nena will come tomorrow, won’t she?
B. He doesn’t deserve our sympathy, does he?
C. We are fortunate, aren’t we?
D. The student who enjoys his work learns quickly, isn’t it?
4. A. She is as good as her sister. C They like Josefa better than I.
B. I love him like a brother. D. They can’t be better than we.
5. A. I met one of the foreign student an hour ago. C. “Who is to blame?”
B. One of them is guilty. D. He was one of those who came late.
6. A. When she saw the stranger fell she utters a loud shriek.
B. He tore everything he took hold of.
C. By this time tomorrow we shall have finished our work.
D. I had reached the shed when it started to rain.
7. A. I heard the news over the radio C. Her request for transfer was granted.
B. She was absent in the meeting. D. She is expected to arrive on Monday.
14
8. A. He fought valiantly.
B. The train goes through some provinces in Central Luzon.
C. All the boats headed for the cove when the tide rose hurriedly.
D. She was rather pretty.
9. A. I have read the book last week C. The visitors arrived yesterday.
B. Jose would have gone home by three o’clock D. The rain has stopped
10. A. The mother as well as her daughter are here again.
B. The teacher, together with her students, is in the library.
C. There are a woman and a child in the room.
D. Either Lily or I am going to attend the party
B. In each group of sentences one does not illustrate any of the following grammar points: variation
from normal word order, parallel structures, subordination for emphasis, use of modifiers for
vividness of expression, economy in expression, and consistent point of view.
1. Which does not observe a consistent point of view?
Set 1.
A. Having no money to spare, you do not know how embarrassing it is for a person at times.
B. Grandmother always boasts of her well-disciplined grandchildren.
C. She can never be a nurse; she passes out at the sight of blood.
D. Little Mara couldn’t wait to open the surprise package she got from Uncle Bert.
Set 2.
A. The farmers have now realized what the government is trying to do to help them.
B. I am usually patient and tolerant with obnoxious people but a person has his limits.
C. Once one has agreed to do a job, he should make every effort to really do it.
D. The government in its desire to uplift the welfare of its people implements various
developmental programs.
2. Which does not observe economy in expression?
Set 1.
A. Two hundred students joined the rally.
B. Last week we had occasion to be the witness of a very interesting incident.
C. The private sector is now cooperating with the government.
D. If I had much money, I would travel the rest of my life.
Set 2.
A. The people started to wave banners and to shout “Mabuhays.”
B. The boys learned to speak with eloquence and to write effectively.
C. He is admired by his friends and feared by his enemies.
D. A fellow who wants to earn some money and to have a little adventure will find working on a
cargo ship exciting.
3. Which does not use parallel structures?
Set 1.
A. The contest organizers gave her the option to go on an all expense-paid tour of Spain or getting a
brand new car.
B. Some of the things she enjoys doing are knitting, sewing and embroidering.
C. It is difficult to work with people who are irresponsible and who cannot be relied upon.
D. Miss Reyes expects her pupils to study diligently and to cooperate in class activities.
Set 2.
A. Read silently and you should keep your eyes from wandering off the page.
B. She doesn’t want to see me or talk to me.
C. The athletes were encouraged by the hopes of their coaches and the support of their countrymen.
D. We knew he was lying but we could not prove it.
15
4. Which does not show variation from normal word order?
Set1.
A. For me there is no excuse C. She dances gracefully.
B. Slowly the men came out of their hiding place. D. One of the paintings he gave to his sister.
Set 2.
A. That book I read when I was in the high school.
B. Fortunate are the persons who register early.
C. Cecile played on quietly, passionately.
D. Patiently she worked out the details of the plan.
5. Which does not use modifiers for vividness of expression?
Set 1.
A. She has a lovely face and a beautiful figure. C. She is as sweet as a rosebud.
B. The jeepney driver was a devil on the loose. D. He works like a machine.
Set 2.
A. The director kept things moving at breakneck speed throughout the performance.
B. The following statistics give, a good idea of the effects of tobacco.
C. To their surprise the damage was not so bad as they expected.
D. It was the first time in my life I had seen Baguio City.
6. Which does not use subordination for emphasis?
Set 1.
A. Seeing that some bystanders were hit by broken bottles, the policemen quickly blew their whistles
and chased the troublemakers.
B. After realizing that it was useless to fight, the girls kissed and made up.
C. When my father saw that I was gasping for breath, he rushed me to the hospital.
D. Sonny was relatively new in school and he though he was the most popular figure in the campus.
Set 2.
A. To be sure that my composition was well organized, I made an outline.
B. At the university, Canny became friendly with Rene, the youngest son of the Prime Minister.
C. As we approached closer to the place we had a strange feeling that we would not be received
cordially.
D. She was famous as a child prodigy and her fame continued throughout her lefe.
TIPS TO HELP YOU COPE
 Read the sentence carefully.
 Check for pronoun errors, if there are none,
check the verbs
 If you find no errors in either verbs or pronouns,
look at adjectives and adverbs.
 Other possible errors include the use of incorrect
idioms and faulty parallelism.
 Be aware of the common grammar and usage errors
16
ENHANCING TEST TAKING SKILLS
Part I – VOCABULARY
In each question, select the word or phrase that most nearly means the same as the underlined word.
Circle the letter of your answer.
1. I’m afraid you can’t see him now, he’s making the rounds.
A. songs sung one after the other C. a kind of dance
B. a series of professional calls on hospital patients D. a knot of people
2. I was itching all over last night so I had to consult a dermatologist.
A. a bone specialist C. a blood specialist
B. a skin specialist D. a kidney specialist
3. Before you start with your hobby of collecting insects, why don’t you consult an entomologist?
A. an insect collector C. a person who is fond of insects
B. a publisher of books on insects D. a specialist in the study of insects
4. Have you ever had an intravenous injection?
A. inside the vein C. near the vein
B. between the veins D. across the veins
5. Manuel is so complacent he does not have a care in the world.
A. satisfied B. uneasy C. excited D. happy
6. She was talking all the time and I couldn’t get a word in.
A. to go out of the room C. to make a correction
B. to stop someone from talking D. to have the chance to speak
7. A bilingual person is one who speaks _______ languages.
A. two B. three C. four D. five
8. Vandalism or any other form of anti-social behavior should not be condoned.
A. condemned B. explained C. overlooked D. permitted
9. The president sees to it that those under him are not compelled to work. He realizes that individuals
who volunteer can accomplish more.
A. persuaded B. lured C. forced D. asked
10. They say that distributing land to the landless is an antidote to the rise of revolutions.
A. a counteraction B. a permissive action C. a related action D. a delaying action
11. Poor Manuel, his girlfriend always leaves him hanging in the balance whenever he talks of marriage.
A. bewildered B. in a state of shock C. rejected D. uncertain
12. Though friends are in accord with one another most of the time, you can also expect them to be in
disagreement at some other time.
A. at war B. thoughtful C. in harmony D. contradicting
13. You will never be able to trace who wrote that controversial article. It bears only a pseudonym.
A. an unpopular name C. an unknown name
B. a fictitious name D. a dead person’s name
14. Juanito’s numerical acuity was displayed during the Math Quiz. He is really sharp when it come to
figures.
A. interest B. involvement C. keenness D. meticulousness
17
15. Ronnie is really the toughest guy in school, he always sticks his neck out to help when someone is in
trouble.
A. to be always present C. to act boldly despite danger
B. to fight anyone D. to call for help
16. Ruby’s skepticism over her colleague’s accomplishments makes her unpopular in the campus.
A. an attitude of doubt C. to indulge in doubting
B. characteristic of a doubtful person D. one who doubts
17. Immediately after her almost tragic accident, she was able to get it all together and acted as though
she didn’t go through one.
A. to gather one’s thoughts C. to move about quickly
B. to stand upright D. to collect one’s composure under pressure
18. I assure you that my dog is docile; it can be trained very easily.
A. manageable B. obedient C. cooperative D. loyal
19. He’s not an effective speaker, he can’t put across his ideas.
A. to make oneself understood C. to get the interest of listeners
B. to deliver a speech properly D. to comprehend what is being said
20. The professor discussed with his class a review of “War and Peace.”
A. a critical article or report on some literary work
B. a judicial re-examination of the decision in a case
C. the process of going over a subject again in study
D. a formal inspection of a military or naval force
Part II – READING
Read each selection carefully and answer the question about it. Circle the letter of your answer.
Selection 1.
White potato production holds much promise as an answer to the recurring rice shortage in much
of Asia.
The potential of the crop as substitute for, or supplement to rice was underscored in a series of
seminars on potato production conducted by foreign and local specialists in Baguio City in Benguet. The
specialists noted that cereal shortage appears to occur to yearly in rice-eating countries of the Western
world. This situation may be traced to the regard given by bread-eating countries to the production of
supplementary food, particularly potato.
In bread-eating countries, white potato is grown extensively and is eaten as a supplement to or a
substitute for wheat bread as the staple food. In rice-eating countries white potato is grown only in a
limited scale and is consumed as vegetable rather than as a principle source of carbohydrate.
The best-selling point of potato as cereal supplement in rice consuming Asian countries is that the
crop is more versatile than rice. For instance, white potato yield per hectare harvest is four to five times
more than rice. With rice, average production is from five to six tons of palay while with white potato,
output is from 20 to 30 tons of tubers.
1. The selection is about
A. the recurring rice shortage in Asia
B. the extensive white potato production in bread eating countries
C. white potato as a practical substitute for rice
D. the limited production of potato in rice-eating countries
2. Which of the following best summarizes the selection?
A. At a meeting of foreign and local specialists, the rice-shortage in Asia was discussed. A practical
solution to this problem, according to them, is substituting white potato for rice.
B. Foreign and local specialists, noting the recurring rice shortage in rice-eating countries, stressed
the potential of white potato as a substitute for rice. Potato harvest per hectare is more than that
of rice.
18
C. An answer to the recurring rice-shortage problem in Asia is white potato production. Whereas in
bread-eating countries, white potato is extensively grown and is a supplement to, or substitute for
wheat bread; in rice-eating countries, it is not and is eaten only as vegetable
D. The rice-shortage problem in rice-eating countries like Asia can be solved by extensive white
potato production. White potato, which is eaten only as vegetable in these countries is a good
substitute for, or supplement to rice, as stressed by foreign and local specialists. An added
advantage of white potato over rice is its greater versatility, with a per hectare yield four or five
times more than rice.
3. According to the selection, which of the following is not true about white potato?
A. It is grown extensively in bread-eating countries.
B. It is eaten as a substitute for rice in rice-eating countries.
C. It is more versatile than rice.
D. It is a supplement to wheat bread in bread-eating countries.
Selection 2
When someone speaks of Troy, he may be thinking of the city in one of three ways – as Troy of
history, as Troy of literature, or as Troy of archaeology.
Historically, we know that the ancient city of Troy, once called Ilium, was located in northwest
Asia Manor, on the banks of the Hellespont. We don’t know the actual age of the city but relics
discovered these have been proven to be as much as 5,000 years old. We know, too, that historical Troy
was a rich and powerful city. It controlled all the commerce of the waters of the area and extended its
power over much of Asia Minor.
The Troy of literature, on the other hand, is the one first celebrated by the Greek poet Homer. His
Odyssey tells the story of Ulysses, one of the heroes of the Trojan War, and his Iliad tells the complete
story of the burning of Troy by the Greeks. Many later writers were inspired by Homer’s stories. They,
in turn, wrote of the beauty of Helen of Troy, of the Greek “gift” to the Trojans of a giant wooden horse,
and of the Greek warriors who won the war and rescued Helen.
In archaeology, Troy is famous in still another way. Heinrich Schliemann’s excavations at
Hisssarlik in 1891 uncovered the rich ruins of a burned city. He thought he had discovered the ruins of
Homer’s Troy. In many later excavations, however, other archaeologists dug down through eight more
cities buried below Schliemann’s. They decided that the city at the seventh level was actually the one of
which Homer wrote.
4. Given below is an incomplete outline for the third paragraph of the selection. Choose the sub-topics
which will best complete the outline.
II. Troy of Literature
A. Greek poet, Homer, first to celebrate Troy
1. Odyssey – story of Ulysses, hero of the Trojan War
2. Iliad – story of the burning of Troy
B. ___________________________________________
1. _________________________________________
2. _________________________________________
3. _________________________________________
A. B. The Trojan War C. B. Stories of Greek Warriors
1. Helen’s beauty 1. Homer’s influence on later writers
2. The Trojan’s gift 2. The beauty of Helen of Troy
3. Helen’s rescue 3. The Greeks gift to the Trojans
B. B. Influence of Homer on Later Writers D. B. The Beauty of Troy
1. The beauty of Helen glorified 1. The beauty of Helen of Troy
2. The story of the giant wooden horse 2. The city’s wealth of power
3. Stories of Greed warriors 3. The courage of its warriors
5. On the whole, the selection is about
A. ancient Troy C. the many ways by which Troy is known
B. the great archeological excavations in Troy D. the rise and fall of Troy
19
6. Given below are sub-topics found in the selection. Choose the one which belongs to the major topic,
Troy of Archeology.
1. Iliad, the story of Troy’s burning
2. Ilium, the ancient city of Troy
3. Heinrich Schliemann’s excavations
4. Odyssey, the story of Ulysses
5. Uncovering the eight more cities
6. The power of the city of Troy
A. 1 and 4 B. 2 and 6 C. 3 and 5 D. 2 and 4
Selection 3
Dear Dr. Strauss:
Under separate cover I am sending you a copy of my report entitled, “The Algernon-Gordon
Effect: A Study of the Structure and Function of Increased Intelligence,” which I would like to have
published.
As you see, my experiments are completed. I have included in my report all of my formulae, as
well as mathematical analysis in the appendix. Of course, these should be verified.
Because of its importance to both you and Dr. Nemur (and need I say to myself, too?) I have
checked and rechecked my results a dozen times in the hope of finding an error. I am sorry to say the
results must stand. Yet for the sake of science, I am grateful for the little bit that I here add to the
knowledge of the function of the human mind and of the laws governing the artificial increase of human
intelligence.
I recall your once saying to me that an experimental failure or the disproving of a theory was as
important to the advancement of learning as a success would be. I know now that this is true. I am sorry,
however, that my own contribution to the field must rest upon the ashes of the work of two men I regard
so highly.
Yours truly,
Charles Gordon
7. What is Charles Gordon?
A. lawyer B. scientist C. teacher D. mathematician
8. The clause “an experimental failure or the disproving of a theory was as important to the
advancement of learning as a success would be” means that
A. only success can advance learning
B. failure slows down advancement of learning
C. advancement of learning is important
D. both experimental failure and success can advance learning
9. The phrase “the results must stand” as found in the third paragraph means that the results of the
experiments
A. must not be changed C. are wrong sometimes
B. must keep changing D. need to be verified several times
10. The last paragraph suggests that his experiment has been
A. disapproved B. successful C. a failure D. incomplete
11. What was Charles feeling while writing the letter?
A. frustrated and sad C. relieved and thankful
B. excited and happy D. nostalgic and resigned
20
Selection 4
The Man He Killed
Had he and I but met
By some old ancient inn,
We would have sat us down to wet
Right many a nipperkin!
But ranged as infantry
And staring face to face.
I shot at him as he at me
And killed him in his place.
I shot him dead because –
Because he was my foe,
Just so: my foe of course he was;
That’s clear enough; although
He thought he’d ‘list, perhaps,
Off-hand – like – just as I –
Was out of work – had sold his traps –
No other reason why.
Yes, quaint and curious war is!
You shoot a fellow down
You’d treat, if met where any bar is,
Or help to half-a-crown.
12. The person speaking in the poem is a
A. bar tender B. murderer C. drunkard D. soldier
13. Which is the best summary of the poem?
A. The two men met in the battlefield and became friends; they met later in an inn.
B. After trying to kill each other in the battlefield, the two men in an inn; they became friends.
C. The two men killed each other in the battlefield; they met in an inn before.
D. The two men were enemies in the battlefield, so when they shot at one another, one of them got
killed; they would have become friends had they met elsewhere.
14. The author must have written the poem to point out the
A. danger of frequenting bars C. foolishness of war
B. need to forgive one’s enemies D. hardship of being jobless
15. What is the best paraphrase of the last stanza?
A. How tragic war is! It makes people not only kill their fellow men but also spend their time and
money in bars.
B. War is a strange thing! In the battlefield one kills a person whom he would treat as a friend
elsewhere.
C. Was should come to an end! It makes one kill another instead of befriending and helping him
D. War is dreadful; it makes you kill even your own friends.
Part III – GRAMMAR
A. Complete each sentence by choosing the answer that will make it correct. Circle the letter of your
answer.
1. He found the book I had lent him __________ on the table
A. lain B. lay C. laid D. lying
2. Her father was one of the businessmen who __________ by the depression
A. are ruined B. is ruined C. were ruined D. is going
21
3. Either French dressing or mayonnaise __________ well with tomatoes.
A. goes B. go C. are going D. is going
4. Lea Salonga sings very well, doesn’t she? __________.
A. Yes, she does. B. No, she does. C. Yes, she doesn’t D. No, she doesn’t
5. __________ every rule there is an exception.
A. In B. To C. For D. Under
6. Anybody __________ it is to his advantage to have a college degree.
A. know B. will know C. knows D. will
7. He hopes you __________ attend his graduation
A. could B. would C. might D.will
8. I could wear my black or brown pair of shoes. Which do you think is __________?
A. good B. better C. well D. best
9. The dog bit Linda __________ hard __________ she cried in pain.
A. such – as B. too – that C. so – that D. such - that
10. When the battle was over, the victorious troops __________ their flag over the city.
A. raised B. raises C. rise D. risen
11. The man was __________ conniving with the hold-up men.
A. accused to B. accused for C. accused of D. accused by
12. “When is Mrs. Chavez planning to retire?”
“Soon, I think. She __________ here for a long time. She’ll probably retire either next year or
the year after that.
A. worked B. had been working C. has been working D. is working
13. “Why did you buy all this sugar and chocolate?”
“I __________ a cake for my friend’s birthday.”
A. make B. am going to make C. will made D. will have made
14. Next week when there __________ a full moon, the ocean tides will be higher.
A. is being B. is C. will D. has been
15. On July 20, 1969, Astronaut Neil Amstrong __________ down into the moon, the first person to set
foot on another celestial body.
A. was stepping B. stepped C. has stepped D. was step
16. Many years of intensive language study are required for non-native speakers to be able to qualify as
interpreters. By the end of this year, Kim __________ English for three years, but he will still need
more training and experience before he masters the language.
A. will be studying C. will have been studying
B. has studied D. has been studying
17. After ten unhappy years, Marissa finally quit the job. She __________ along with her boss for a long
time before she finally decided to look for a new position.
A. hadn’t been getting B. isn’t getting C. hasn’t been getting D. didn’t get
18. A minor earthquake occurred at 2:07 a.m. on January 3. Most of the people in the village
__________ at that time and didn’t even know it had occurred until the next morning.
A. slept B. had slept C. were sleeping D. sleep
19. Gina’s eyes burned and her shoulder ached. She __________ at the computer for five straight hours.
Finally she took a break.
A. is sitting B. has been sitting C. was sitting D. had been sitting
22
20. Inah and Jojie were mischievous children. They __________ tricks on their teachers which always
got them into a lot of trouble.
a. could play B. would play C. play D. played
21. You have to pay extra if you take too __________ with you.
A. much luggage B. many luggages C. much luggages D. many luggage
22. At the news conference, several reporters didn’t get clear answers to __________ questions.
A. theirs B. their C. his and hers D. his and her
23. Two-thirds of my __________ from the province.
A. classmate is B. classmate are C. classmate is D. classmates are
24. If I could speak French, I __________ next year studying in France.
A. would spend B. would have spent C. has spent D. will spend
25. If I __________ the same problems as you had as a child, I might not have succeeded in life as well
as you have.
A. have B. would have C. had D. should have
26. By measuring changing conditions in the atmosphere, __________ general weather patterns.
A. meteorologists who are predicting C. predicting meteorologists
B. meteorologists were predicted D. meteorologists can predict
27. Not until the end of the nineteenth century, __________ become a scientific discipline.
A. plant breeding had B. did plant breeding C. plant breeding have D. have plant breeding
28. The cerebral cortex is __________ where the process of remembering faces takes place.
A. the area is brained B. the area of the brain C. and a brain area D. brain area
29. By the end of the nineteenth century, Thomas Edison had invented the first practical light bulb,
__________.
A. a source of cheap electrical light C. a source light cheap electrically
B. the light of electricity cheap source D. light with cheap electrically source
30. __________ about babies’ feelings is inferred from their expressions.
A. What we know B. To be known by us C. Knowing D. Known
31. __________, communities are formed in a variety of ways.
A. Created a division of labor C. Create a division of labor
B. To create a division of labor D. Creation of division of labor
32. The condition necessary __________ this project have not been meet.
A. for the complete B. of completion C. for completion of D. of complete
33. __________ are found in virtually every country in the world.
A. Fruit flies and mosquitoes. C. When fruit flies and mosquitoes
B. Now that fruit flies and mosquitoes D. Fruit flies and mosquitoes
34. Successful salespeople __________ and understand the needs of the market.
A. products are thoroughly known C. thoroughly know their products are
B. know their products thoroughly D. their products are thoroughly known
35. The number of members of the executive board in a big corporation is fixed by the bylaws
__________ by the president.
A. nevertheless B. despite C. instead D. not
36. Lasers __________ steel by focusing an intense beam on the metal.
A. cutting B. cut C. to cut D. of cutting
37. __________ common nuclear reactions, cold fusion does not require a high temperature.
A. Alike B. It is unlikely C. It is not like D. Unlike
23
38. Native American people arrived on the North American continent __________.
A. since B. for C. before D. ahead
39. The ways of traveling __________ dramatically since the late nineteenth century.
A. will have changed B. has changed C. have changed D. will change
40. How many of us __________ over complicated changes in the law!
A. not frustrated C. have not become frustrated
B. not become frustrated D. is not frustrated
41. A promissory note __________ anything without the trust deed.
A. does mean B. means C. have not mean D. may not mean
42. Dairy farming is __________ leading agricultural activity in the United States.
A. a B. at C. then D. none
43. Although thunder and lightning are produced at the same time, light waves travel faster, __________
so we see the lightning before we hear the thunder.
A. than sound waves do C. do sound waves
B. than sound waves are D. sound waves
44. Beef cattle __________ of all livestock for economic growth in certain geographic regions.
A. the most are important C. the most important are
B. are the most important D. that are the most important
45. The discovery of the half tone process in photography in 1881 made it __________ photographs in
books and newspapers.
A. the possible reproduction C. the most important are
B. possible to reproduce D. that are the most important
46. Beginning in the Middle Ages, composers of Western music used a system of notating their
compositions __________ be performed by musicians
A. will B. that C. and when to D. so they could
47. Civil Rights are the freedoms and rights __________ as a member of a community, state or nation.
A. may have a person C. a person may have
B. may have a person who D. and a person may have
48. Richard Wright enjoyed success and influence __________ among Black American writers of his era.
A. were unparalleled B. are unparalleled C. unparalleled D. the unparalleled
49. Franklin D. Roosevelt was __________ the great force of radio and the opportunity it provided for
taking government policies directly to the people.
A. as the first President he understood fully C. the first President to fully understand
B. the President fully understand D. the first President to understand fully
50. During the late fifteenth century, __________ of the native societies of America had professions in
the fields of arts and crafts.
A. only a few B. a few but C. few but only D. a few only
B. Write the number that corresponds to the word or phrase which makes the sentence incorrect.
Write number 5 if the sentence has no error.
_____ 1. You are the very-person who I saw.
1 2 3 4
_____ 2. She spoke clear and made a good impression on the audience.
1 2 3 4
_____ 3. We do our shopping in Makati because of their big stores there.
1 2 3 4
_ ____ 4. I think a hundred pesos are too much for that toy.
1 2 3 4
24
_____ 5. Exploding firecrackers during the New Year are a time-honored tradition in our country.
1 2 3 4
_____ 6. If I was you I would think twice before resigning.
1 2 3 4
_____ 7. If he had only knew that the U.S. would invade Panama he would have been on the alert.
1 2 3 4
_____ 8. Since its establishment in 1982 the company is very progressive.
1 2 3 4
_____ 9. Ben, together with Mon and Ronnie have been chosen to play for the national team.
1 2 3 4
_____ 10. Rizza’s mother scolded her when she break the imported vase.
1 2 3 4
_____ 11. My teacher and friend, Mr. Torres are with my groupmates
1 2 3 4
_____ 12. The security guard assured us that everything will be taken cared of.
1 2 3 4
_____ 13. We should not allow aliens to exploit our natural resources.
1 2 3 4
_____ 14. One of the puppies looks different from it’s parents.
1 2 3 4
_____ 15. “You shouldn’t meddled in the controversy,” Aunt Sophia said.
1 2 3 4
_____ 16. Each student who attends the orientation last Saturday was requested to fill out a personal data
sheet. 1 2 3 4
_____ 17. When the policeman saw the robber, he handcuffed the man who wished he has not run that
direction. 1 2 3 4
_____ 18. Although Dan had known the combination to the lock, he never told anyone
1 2 3 4
_____ 19. Please notify we committee members of the next meeting.
1 2 3 4
_____ 20. Ask them or us anything you want to know you want to know about the computer.
1 2 3 4
_____ 21. She is usually complementary to whomever she is with.
1 2 3 4
_____ 22. Either of the girls might have a chance to sing their song on television
1 2 3 4
_____ 23.Every man, woman, and child in the Philippines are guaranteed certain rights.
1 2 3 4
_____ 24. I don’t know which is worst, packing to go away or unpacking.
1 2 3 4
_____ 25. I’d advise you to follow your doctor’s advise.
1 2 3 4
_____ 26. In some species of fish, such the three-spined stickleback, the male, not the female, performs
1 2 3
the task of caring for the young.
4
_____27. When she retires in September 1989, tennis champion Christine Evert was the most famous
1 2 3
woman athlete in the United States.
4
_____ 28. The ancient Romans used vessels equipped with sails and banks of oars to transporting their
1 2 3 4
armies.
_____29 Dinosaurs are traditionally classified as cold-blooded reptiles, but recent evidence based on
1 2
eating habits, posture, and skeletal structural suggests some may have been warm-blooded.
3 4
25
_____30 In human beings, as in other mammal hairs around the eyes and ears and in the nose, prevent
1 2 3
dust, insects, and other matter from entering these organs.
4
_____31. The works of the author Herman Melville are literary creations of a high order, blending fact,
1 2 3
fiction, adventure, and subtle symbolic.
4
_____32. Each chemical element is characterized to the number of protons that an atom of that element
1 2 3
contains, called its, atomic number.
.4
_____33. The body structure that develop in birds over millions of well designed for flight, being both
1 2 3
lightly in weight and remarkably strong.
4
_____34. From 1905 to 1920, American novelist Edith Wharton was at the height of her writing career,
1 2 3
publishing of her three most famous novels.
4
_____ 35. In the early 20th
century, there was considerable interesting among sociologists in the fact that
1 2
in the United States the family was losing its traditional roles.
3 4
_____36. Although pure diamond is colorless and transparent, when contaminated with other material it
1 2 3
may appear in various color ranging from pastels to opaque black.
4
_____37. Comparative anatomy is concerned to the structural differences among animal forms.
1 2 3 4
_____38. A seismograph records oscillation of the ground caused by seismic waves, vibrations that travel
1 2
from its point of origin through the Earth or along its surface.
3 4
_____39. Electric lamps came into widespread use during the early 1900 and have replaced other type of
1 2
fat, gas, or oil lamps for almost every purpose.
3 4
_____40. Emily Dickinson, among the greatest woman poets in the English language, died with all of her
1 2 3
poems unpublished, except for seven that appeared in the publications of limited circulation.
4
_____41. Protecting coral reefs is difficult because some of the corals are very fragile; even the touch of
1 2 3 4
the diver’s hand can kill it.
_____42. To improvise effectively, a musician must thorough understand the conventions of a given
1 2 3 4
musical style.
_____43. Leaves are believed to be one of the best substance to form compost piles.
1 2 3 4
_____44. Wood is an excellent resource for heating homes cooking food, and build houses.
1 2 3 4
_____45. River water pollution is often indicator by algae distribution.
1 2 3 4
_____46. Modern farms are much larger than that of former times.
1 2 3 4
_____47. It is often not until the end of their lives that famous people receive the recognize they deserve.
1 2 3 4
_____48. Usually the climate in mountainous areas becomes much windy at higher altitudes.
1 2 3 4
26
_____49. A physical chemist is required to function simultaneously as a physicist, chemist and
1 2 3 4
mathematician.
_____50. A baby gorilla is a shy, friendly animal that like attention.
1 2 3 4
THE TRADITIONAL CONCERNS: WORD CHOICE AND GRAMMAR
A. Sentence Fragment. A sentence expresses a logically complete idea. If the idea is not
complete – if your reader is left wondering what you mean – you probably have omitted
some essential element.
? She spent her first week on the job as a researcher. Compiling information
from digests and journals.
? Because the operator was careless. The new computer was damaged.
B. Comma Splice. Two complete ideas (independent clauses), which should be separated by a
period or a semicolon, are incorrectly joined by a comma.
? Sarah did a great job, she was promoted.
? This is a new technique, therefore, some people mistrust it.
C. Run-on Sentence. A sentence that crams, too many ideas without needed breaks or pauses.
? The hourglass is more accurate than the water clock because water in a water
clock must always be at the same temperature to flow at the same speed since
water evaporates and must be replenished at regular intervals, thus being less
effective than the hourglass for measuring time.
D. Faulty Coordination. Ideas of equal importance are joined, within simple or compound
sentences, with coordinating conjunctions: and, but, or, nor, for, so, and yet.
? Joseph had a drinking problem and he dropped out of school.
? I will try and help you.
? The climax in jogging comes after a few kilometers and I can no longer feel
stride after stride and it seems as if I am floating and jogging becomes almost a
reflex and my arms and legs continue to move and my mind no longer has to
control their actions.
E. Faulty Subordination. Proper subordination shows that a less important idea is dependent
on a more important idea using subordinating conjunctions: because, so, if, unless, after,
until, since, while, as and although.
? Television viewers can relate to an athlete they idolize and they feel obliged to
but the product endorsed by their hero.
? This teacher is often late for work, and he writes illogical reports, and he is a
poor manager, and he should be fired.
? This job which I took when I graduated from college, while I waited for a better
one to come along, which is boring, where I’ve gained no useful experience, make
me anxious to quit.
F. Faulty Agreement – Subject and Verb. The subject should agree in number with the verb.
In more complicated sentences – those in which the subject is separated from its verb by
other words – we sometimes lose track of the subject-verb relationship.
? The lion’s share of diesels are sold in Europe.
? A system of line extend horizontally to form a grid.
? Each of the crew members were injured.
? Everyone in the group have practiced long hours.
? The committee meet weekly to discuss ness business.
G. Faulty Agreement – Pronoun and Referent. A pronoun can make sense only if it refers to
a specific noun (its referent or antecedent), with which it must agree in gender and number.
? Anyone can get their degree form that college.
? Each candidate described her plans in details.
27
H. Faulty or Vague Pronoun Reference. Whenever a pronoun is used, it must refer to one
clearly identified referent; otherwise the message will be confusing.
? Rolly told Ramon that his wife loves him.
? He drove away from his menial job, boring lifestyle, and damp apartment,
happy to be leaving it behind.
? The problem with our defective machinery is compounded by the operator’s
incompetence. That annoys me!
I. Faulty Pronoun Case. A pronoun’s case (nominative, objective, or possessive) is
determined by its role in a sentence: as subject, object, or indicator of possession.
? Whom is responsible to who?
? The debate was between Marina and I.
? Us teacher are accountable for our decisions.
? A group of we teachers will fly to the convention.
J. Faulty Modification. A sentence’s word order (syntax) helps determine its effectiveness
and meaning.
? Dialing the phone, the cat ran out the open door.
? While walking, a cold chill ran through my body.
? Gina typed another memo on our computer that was useless.
? She volunteered immediately to help the landslide victims in Leyte.
K. Faulty Parallelism. To reflect relationships among items of equal importance, express them
in identical grammatical form.
? We here highly resolve … that government of the people, which the people
created and maintain, serving the people shall not perish from the earth.
? The new teacher is enthusiastic, skilled, and you can depend on her.
? In her new assignment, she felt lonely and without a friend.
? She sleeps well, jogs daily, as well as eating high-protein foods.
L. Sentence Shifts. Shifts in point of view damage coherence.
? When you finish the jog, one will have a sense of pride.
? One should sift the flour before they make the pie.
? He delivered the plans for the apartment complex, and the building site was also
inspected by him.
? She delivered the blueprints, inspected the foundation, wrote her report, and
takes the afternoon off.
? Unscrew the valve and then steel wool should be used to clean the fitting.
? Jim wonders, if he will get the job and will he like it?
M. Effective Punctuation. Punctuation marks are like road signs and traffic signals that give a
simple way of making one be understood. Compare the two letters below.
Dear Jack,
I want a man who knows what love is
all about. You are generous, kind, and
thoughtful. People who are not like
you admit to being useless and inferior.
You have ruined me for other men. I
yearn for you. I have no feelings
whatsoever when we’re apart. I can be
forever happy – will you let me be
yours?
Jill
Dear Jack,
I want a man who knows what love is.
All about you are generous, kind, and
thoughtful people who are not like you.
Admit to being useless and inferior.
You have ruined me. For other men I
yearn! For you. I have no feelings
whatsoever. When we’re apart I can
be forever happy. Will you let me be?
Yours,
Jill
28
Practice Exercises
A. The following letter to the editor of a newspaper has twelve errors in subject-verb agreement and
pronoun-antecedent agreement. Find and correct the errors. Use forms that are correct for
formal English.
To the Editor, The Times
Many parts of our once-beautiful city is starting to look like mini garbage dumps. You will recall that
legislation requiring recycling within the city limits were passed last year, and the mayor and other
local politicians encourages us to recycle, but in my apartment complex there is no bins for recycling.
The result is that people take no responsibility for his own actions, and everyone tosses their trash and
recyclables (glass, plastic bottles, cans, etc) right in with the food that is being thrown away. Neither
the manager of the complex nor the owners of the building has bought any new containers for the
items that are supposed to be recycled. So what else can everyone do but mix their trash together?
Either the manager or the owners is responsible for breaking the law here. Not us! Meanwhile,
trashcans in the downtown area is overflowing with garbage, and vacant lots all around the city is
littered with soda cans, broken glass, and paper. The owner and publisher of your newspaper, Mr.
Romeo Gomez, have always been a supporter of clean environment. I urge your paper to take
leadership in solving this problem.
B. Read the following paragraph about the Judge Crater mystery. Correct three errors I
parallelism with active or passive voice
On the evening of August 6, 1930, Judge Force Crater, a wealthy, successful, and
good-looking New York lawyer disappeared without a trace. Earlier in the evening he had
been seen with friends at a Manhattan restaurant. At 9:10 P.M. he left the restaurant, hailed a
taxi, and was driven away. No one ever saw or heard from him again. It was ten days before
he was even reported missing. On August 16, his wife called his courthouse, the secretary
was asked about his whereabouts, and learned that he was probably off on political business.
This news reassured Mrs. Crated somewhat, but when he still hadn’t turned up by August 26,
a group of his fellow judges started an investigation. A grand jury was convened, but its
members could not come to any conclusion as to what had happened to Judge Crater. They
theorized that the judge might have developed amnesia, run away voluntarily, or been a
crime victim. He wife disagreed with the first two possibilities, holding that he had been
murdered by someone in the Tammany Hall organization, the political machine that
controlled New York City at the that time. The mystery remains unsolved to this day. He
could have been killed by a Tammy hall hiree, a girlfriend could have murdered him, or
kidnapped by an organized crime group. He might in fact have suffered from amnesia, or he
might have planned his own disappearance. Reports of Judge Crater sightings have
continued to surface over the last sixty years.
Words and Expressions Commonly Misused. Many words and expressions are not so much
bad English as bad style, the commonplaces of careless writing.
 All right. Idiomatic in familiar speech as a detached phrase in the sense “Agreed,” or
“O.K.”
 Alternate, Alternative. The words are not always interchangeable as nouns or
adjectives. The first means every other one in a series; the second, one of two
possiblilities.
 Among, Between. When more than two things or persons are involved, among is usually
called for. When, however, more that two are involved but each is considered
individually, between is preferred.
 Anticipate. Use expect in the sense of simple expectation
 Anybody. In the sense of “any person,” not to be written as two words, Any body means
“any corpse.”
 Anyone. In the sense of “anybody,” written as one word. Any one means “any single
person.”
 As to whether. Whether is sufficient.
 As yet. Yet nearly always is as good, if not better.
 Careless. The dismissive “I couldn’t care less” is often used with the shortened “not”
mistakenly omitted: “ I could care less.”
29
 Certainly. Used indiscriminately by some speakers much as other use very, in an attempt
to intensify any and every statement.
 Consider. Not followed by as when it means “believe to be.”
 Contact. As a transitive verb, the word is vague and self-important. Do not contact
people; get in touch with them, look them up, phone them, find them, or meet them.
 Cope. An intransitive verb used with with. In formal writing, one doesn’t “cope,” one
“copes with” something or somebody.
 Disinterested. Means “impartial.” Do not confuse it with uninterested which means
“not interested in.”
 Each and every one. Avoid, except in dialogue.
 Effect. As a noun, means “result”; as a verb, means “to bring about,” “to accomplish”
(not to be confused with affect, which means “to influence”)
 Enthuse. An annoying verb growing out of the noun enthusiasm. Not recommended.
 Factor. A hackneyed word; the expressions of which is a part can usually be replaced by
something more direct and idiomatic
 Farther, Further. Farther serves best as distance word, further as a time or quantity
word
 Finalize. A pompous, ambiguous verb.
 Hopefully. This once-useful adverb meaning “with hope” has been distorted and is now
widely used to mean “I hope” or “it is to be hoped.”
 However. Avoid starting a sentence with however when the meaning is “nevertheless.”
The word usually serves better when not in first position.
 Imply, Infer. Not interchangeable. Something implied is something suggested or
indicated, though not expresses. Something inferred is something deduced from evidence
at hand.
 In regard to. Often wrongly written in regards to. But as regards is correct, and means
the same thing.
 Insightful. The word is suspicious overstatement for “perceptive.”
 Irregardless. Should be regardless.
 -ize. Do not coin verbs by adding this tempting suffix. Many good and useful verbs do
end in –ize: summarize, harmonize, fertilize. But there is a growing list of abominations:
prioritize, utilize, personalize, finalize.
 Kind of. Except in familiar style, not to be used as a substitute for rather or something
like.
 Like. Not to be used for the conjunction as. Like governs nouns and pronouns; before
phrases and clauses the equivalent word is as.
 Nice. A shaggy, all-purpose word, to be used sparingly in formal composition.
 Ongoing. A mix of “continuing” and “active” and is usually superfluous.
 One of the most. There is nothing wrong with the grammar; the formula is simply
threadbare.
 Prestigious. Often an adjective of last resort. It’s in the dictionary, but that doesn’t
mean you have to use it.
 Respective, Respectively. These words may usually be omitted with advantage.
 Secondly, thirdly, etc. Unless you are prepared to begin with firstly, and defend it
(which will be difficult), do not prettify numbers with –ly.
 Thanking you in advance. This sounds as if the writer meant, “It will not be worth my
while to write to you again.”
 The foreseeable future. A cliché, and a fuzzy one.
 The truth is … The fact is … A bad beginning for a sentence. If you feel you are
possessed of the truth, or of the fact, simple state it. Do not give advance billing.
 Very. Use this word sparingly. Where emphasis is necessary, use words strong in
themselves.
30
C. Correct the misused words and expressions in the sentences below.
1. An agreement was reached among the two companies to work on the same project as equal partners.
2. The schedule has not been finalized yet.
3. The office managers considered the new policy as rigid and unreasonable.
4. The case of the dismissed employee should be a lesson to each and everyone of us.
5. After considering the applicant’s qualifications, I can say that he is a man who is knowledgeable
and well-trained for the job.
6. The project is going to be finished as scheduled. More importantly, the company is able to save
one-third of the original budget.
7. Irregardless of the problems encountered at the project site, we implemented the plan as scheduled.
8. This is a nice proposal to improve the flow of communications within the company.
9. The office cannot authorize nor justify overtime work rendered on Sundays.
10. Our company needs to prioritize the requests made by each department regarding office supplies.
11. I would like to assure you that the project will be completed as scheduled. Personally, I believe that
all the problems have been addressed adequately.
12. The project director stated that no new projects will be approved until the end of the year.
13. The truth is the workers are overworked and underpaid.
14. The proposal is so interesting that the company agrees to meet with the people concerned.
15. Most all the time, the reports of the engineers do not include the day-to-day progress of work in the
job site.
D. Filipinism is language of the Philippine English variety. As such, they may be understood
by Filipino speakers but considered unidiomatic by native English speakers. How do you
make the following Filipinisms more idiomatic?
1. I drink medicine daily.
2. He declared his love for her.
3. We called a police.
4. Will you pass first?
5. I am vacant.
6. I dress up before breakfast.
7. My watch is advanced/behind.
8. The maid will serve us.
9. Come already, we shall be late.
10. The student’s pension is enough for his needs.
11. I arrange my bed every morning.
12. Maria has a bad custom of copying.
13. Open the light.
14. Repeat the sentence again.
15. You are becoming fat.
16. She is my co-teacher.
17. How is your Mrs.?
18. I am finished.
19. I’ll tell you to my mother.
20. How is life getting along with you?
31
READING IN THE CONTENT AREAS
I. Read each excerpt and choose the best answer to each question that follows.
American History
(An excerpt, by Judith Ortiz-Cofer, 1992)
5
10
15
The day Mr. DePalma came out into the cold and asked us to line up in front of him
was the day that President Kennedy was shot. Mr. DePalma, a short, muscular man with
slicked-down black hair, was the science teacher, P.E. coach, and disciplinarian at P.S. 13.
He was the teacher to whose homeroom you got assigned if you were a troublemaker, and
the man called out to break up playground fights, and to escort violently angry teenagers to
the office. And Mr. DePalma was the man who called your parents in for a “conference.”
That day, he stood in front of two rows of mostly black and Puerto Rican kids, brittle
from their efforts to “keep moving” on November day that was turning bitter cold. Mr.
DePalma, to our complete shock, was crying. Not just silent adult tears, but really sobbing.
There were a few titters from the back of the line where I stood shivering.
“Listen,” Mr. DePalma raised his arms over his head as if he were about to conduct an
orchestra. His voice broke, and he covered his face with his hands. His barrel chest was
heaving. Someone giggled behind me.
“Listen,” he repeated, “something awful has happened.” A strange gurgling came
from his throat, and he turned around and spat on the cement behind him.
“Gross,” someone said, and there was a lot of laughter.
“The President is dead, you idiots. I should have known that wouldn’t mean anything
to a bunch of losers like you kids. Go home.”
***********
1. Where does the action of the story take place?
A. in the gym of a community center C. on the front steps of a federal courtroom
B. on the playground of a public school D. in the yard of a federal prison
2. Mr. DePalma might best be described as
A. softhearted B. funny C.conceited D. strict
3. Mr. DePalma cried because he was
A. hurt that people were making fun of him
B. upset about the death of the president
C. sick to his stomach and unable to breathe
D. frightened that war was about to be declared
4. How does Mr. DePalma feel about the speaker and her friends?
A. indifferent B. proud C. affectionate D. disgusted
5. The author develops the character of Mr. DePalma in all of the following ways except
A. revealing what Mr. DePalma says to other characters
B. disclosing what Mr. DePalma secretly thinks
C. showing how Mr. DePalma interacts with others
D. describing what Mr. DePalma looks like
32
Suburban Prophecy
By Howard Nemerov, 1977
5
10
On Saturday, the power-mowers whine
Begins the morning. Over this neighborhood
Rises the keening, petulant voice, begin
Green oily teeth to chatter and munch the cud.
Monsters, crawling the carpets of the world,
Still send from underground against your blades
The roots of things battalions green and curled
And tender, that will match your blades with blades
Till the revolted throats shall strangle on
The tickle of their dead, till straws shall break
Crankshafts like camels, and the sun go down
On dinosaurs in swamps. A night attack
Follows and by the time the Sabbath dawns
All armored beasts are eaten by their lawns.
**********
6. In the first stanza, the poet uses words such as whine, voice, teeth, chatter, and munch to suggest that
the power-mowers are
A. alive B. in need of repair C. like cows D. very powerful
7. The imagery in the first stanza most appeals to the reader’s sense of
A. sight B. touch C. hearing D. taste
8. The phrase “your blades” in line 8 refers to
A. grass B. gardeners C. lawnmowers D. monsters
9. The action of the poem takes place in the time period of
A. an afternoon B. a morning C. twenty-four hours D. a week
10. The events and imagery of the second stanza create an atmosphere in which
A. suburbanites use their lawnmowers against each other
B. the people pushing the mowers are completely exhausted
C. the smell of freshly cut grass lingers in the air
D. lawns and lawnmowers are battling
11. Throughout the poem, Nemerov creates an interesting image of lawnmowers by comparing them to
A. animals B. other machines C. people D. plants
12. The poet is ridiculing
A. suburbanites who turn into monsters C. lawnmowers and other modern machines
B. suburbanites who keep perfect lawns D. realtors who sell house without lawns
13. In one description, Nemerov makes a comparison using simile and alliteration. Which of the
following descriptions represents both of these techniques?
A. “crawling the carpets of the world”
B. “Green oily teeth to chatter and much the cud”
C. “Crankshafts like camels”
D. “and the sun go down/ on dinosaurs in swamps”
33
“Beauty Marks”
Review of Smile
Music by Marvin Hamilsch
Book and Lyrics by Howard Ashman
by William A. Henry III,
Time, December 8, 1986
5
10
15
This season’s first four new American musicals all closed the seek they opened,
continuing a daunting three-year run of almost unrelieved financial failure for what used to
be Broadway’s mainstay. Staged with varying degrees of artistry, the ill-fated shows shared
one disabling presumption: musicals must be “about” something beyond melody and
romance … All suffocated under the weight of their ambition.
One might expect the same fate to befall Smile, adapted from a 1975 Michael Ritchie
film that satirized beauty pageants. The narrative, centering on girls who are strangers,
inevitably lacks complex relationships and love interest. Moreover, it is difficult to write a
parody much funnier than the real Miss America proceedings. And it is hard to keep
audiences interested in the climax – which entrant will win – after repeatedly telling them it
shouldn’t matter. Curiously, Smile works. It is a swift-paced, skillfully performed and
thoroughly professional entertainment that balances amusement at the shallow ambitions of
the characters with respect for the depth of their feelings. Composer Marvin Hamlisch (A
Chorus Line) and Author-Lyricist Howard Ashman (Little shop of Horrors) have written
touching songs for the stars, Anne Marie Bobby as a sweet, awkward A student who realizes
she is out of her element at the pageant and Jodi Benson as a wanderer who is prematurely
wise in the ways of selling herself, including a talent-show “dramatic reading” that turns
into a strip-tease. Smile may not be a landmark, but it is a pleasure.
**********
14. According to the critic, musicals were, at the time,
A. as popular as ever C. unsuccessful on Broadway
B. gaining popularity each year D. not as popular as films
15. Lines 5 state that “All suffocated under the weight of their ambition.” The critic includes this
descriptive language to suggest that
A. the directors and actors were not experienced
B. the musical relied too much on light humor
C. producers were too concerned with winning awards
D. the musicals tried to make a statement as well as entertain
16. The critic states that the musical “balances amusement at the shallow ambitions of the characters with
respect for the depth of their feelings” (lines 12-13). This expresses a contrast between the audience’s
A. feelings about Miss America and patriotism
B. depth of understanding of the two main characters
C. mixed emotions about the characters’ struggles
D. favorable opinion of Smile
17. The critic believes the music is
A. too humorous for the theme C. appropriate for the characters
B. shallow and without purpose D. outstanding and destined to be popular
18. Which of the following statements describes the critic’s opinion of Smile?
A. Although Smile is not a great musical, it is enjoyable.
B. Smile is an example of why musicals are no longer popular.
C. Smile unjustly criticizes the Miss America pageant.
D. Since Smile is about women, it will surely be popular.
19. According to the experience of Broadway sows for three years running, the type of production that
would have been most likely to fail would be a
A. comedy about marriage C. one-woman concert
B. drama about Nazi Germany D. musical about a person’s personal triumph
20. The statement “Curiously, Smile works” reveals the critic’s
A. total disgust C. lack of faith in the audience
B. dislike of the musical D. surprise at the musical’s success

More Related Content

What's hot

1st quarter exam english 8
1st quarter exam english 81st quarter exam english 8
1st quarter exam english 8Myline Unggos
 
Grade 12 Module : Hope 3 Recreational Activities
Grade 12 Module : Hope 3 Recreational ActivitiesGrade 12 Module : Hope 3 Recreational Activities
Grade 12 Module : Hope 3 Recreational ActivitiesNicole Angelique Pangilinan
 
Use Conventions in Citing Sources
Use Conventions in Citing SourcesUse Conventions in Citing Sources
Use Conventions in Citing SourcesMsCarestigoy
 
Period of Emergence Game
Period of Emergence GamePeriod of Emergence Game
Period of Emergence GameMarjorie Calar
 
Voice of the Verbs
Voice of the VerbsVoice of the Verbs
Voice of the Verbsfeueacmrq
 
Banghay aralin sa filipino( 9 lessons)ikatlong markahan
Banghay aralin sa filipino( 9  lessons)ikatlong markahanBanghay aralin sa filipino( 9  lessons)ikatlong markahan
Banghay aralin sa filipino( 9 lessons)ikatlong markahanlovelyjoy ariate
 
Maikling Kuwento ni Mabuti, gabay sa pagbasa
Maikling Kuwento ni Mabuti, gabay sa pagbasaMaikling Kuwento ni Mabuti, gabay sa pagbasa
Maikling Kuwento ni Mabuti, gabay sa pagbasaNeri Zara
 
Japanese and chinese traditions
Japanese and chinese traditionsJapanese and chinese traditions
Japanese and chinese traditionsChristine David
 
SAMPLE: Grade 10 - APA Bibliography Detailed Lesson Plan
SAMPLE: Grade 10 - APA Bibliography Detailed Lesson PlanSAMPLE: Grade 10 - APA Bibliography Detailed Lesson Plan
SAMPLE: Grade 10 - APA Bibliography Detailed Lesson PlanMay Andrea Francia
 
READING & WRITING SKILLS week 1.pptx
READING & WRITING SKILLS week 1.pptxREADING & WRITING SKILLS week 1.pptx
READING & WRITING SKILLS week 1.pptxEllaMaeMamaedAguilar
 

What's hot (20)

Sentence Patterns Fact/Worksheet
Sentence Patterns Fact/WorksheetSentence Patterns Fact/Worksheet
Sentence Patterns Fact/Worksheet
 
Tayutay ppt
Tayutay pptTayutay ppt
Tayutay ppt
 
1st quarter exam english 8
1st quarter exam english 81st quarter exam english 8
1st quarter exam english 8
 
ENGLISH Grade 8 Q3 L3
ENGLISH Grade 8 Q3 L3ENGLISH Grade 8 Q3 L3
ENGLISH Grade 8 Q3 L3
 
Grade 12 Module : Hope 3 Recreational Activities
Grade 12 Module : Hope 3 Recreational ActivitiesGrade 12 Module : Hope 3 Recreational Activities
Grade 12 Module : Hope 3 Recreational Activities
 
Elements of poetry
Elements of poetryElements of poetry
Elements of poetry
 
Use Conventions in Citing Sources
Use Conventions in Citing SourcesUse Conventions in Citing Sources
Use Conventions in Citing Sources
 
Period of Emergence Game
Period of Emergence GamePeriod of Emergence Game
Period of Emergence Game
 
Grading sh-learners
Grading sh-learnersGrading sh-learners
Grading sh-learners
 
Voice of the Verbs
Voice of the VerbsVoice of the Verbs
Voice of the Verbs
 
Banghay aralin sa filipino( 9 lessons)ikatlong markahan
Banghay aralin sa filipino( 9  lessons)ikatlong markahanBanghay aralin sa filipino( 9  lessons)ikatlong markahan
Banghay aralin sa filipino( 9 lessons)ikatlong markahan
 
Idyoma
IdyomaIdyoma
Idyoma
 
Maikling Kuwento ni Mabuti, gabay sa pagbasa
Maikling Kuwento ni Mabuti, gabay sa pagbasaMaikling Kuwento ni Mabuti, gabay sa pagbasa
Maikling Kuwento ni Mabuti, gabay sa pagbasa
 
Natural and inverted order of sentences
Natural and inverted order of sentencesNatural and inverted order of sentences
Natural and inverted order of sentences
 
Literary Genre quiz
Literary Genre quizLiterary Genre quiz
Literary Genre quiz
 
Japanese and chinese traditions
Japanese and chinese traditionsJapanese and chinese traditions
Japanese and chinese traditions
 
SAMPLE: Grade 10 - APA Bibliography Detailed Lesson Plan
SAMPLE: Grade 10 - APA Bibliography Detailed Lesson PlanSAMPLE: Grade 10 - APA Bibliography Detailed Lesson Plan
SAMPLE: Grade 10 - APA Bibliography Detailed Lesson Plan
 
Talumpati.pptx
Talumpati.pptxTalumpati.pptx
Talumpati.pptx
 
READING & WRITING SKILLS week 1.pptx
READING & WRITING SKILLS week 1.pptxREADING & WRITING SKILLS week 1.pptx
READING & WRITING SKILLS week 1.pptx
 
English 9 Q1 Week 1
English 9 Q1 Week 1English 9 Q1 Week 1
English 9 Q1 Week 1
 

Viewers also liked

A review-on-principles-of-teaching
A review-on-principles-of-teachingA review-on-principles-of-teaching
A review-on-principles-of-teachingZanita Corullo
 
Science k-12-curriculum-guides
Science k-12-curriculum-guidesScience k-12-curriculum-guides
Science k-12-curriculum-guidesZanita Corullo
 
K to-12-mathematic-curriculum-guide-grade-1
K to-12-mathematic-curriculum-guide-grade-1K to-12-mathematic-curriculum-guide-grade-1
K to-12-mathematic-curriculum-guide-grade-1Hannah Karylle Dacillo
 
English curriculum guide grades 1 10 december 2013
English curriculum guide grades 1 10 december 2013English curriculum guide grades 1 10 december 2013
English curriculum guide grades 1 10 december 2013lambert manansala
 
K to 12 Mathematics Curriculum Guide for Grades 1 to 10
K to 12 Mathematics Curriculum Guide for Grades 1 to 10K to 12 Mathematics Curriculum Guide for Grades 1 to 10
K to 12 Mathematics Curriculum Guide for Grades 1 to 10Dr. Joy Kenneth Sala Biasong
 

Viewers also liked (8)

Weathertheme[1]
Weathertheme[1]Weathertheme[1]
Weathertheme[1]
 
A review-on-principles-of-teaching
A review-on-principles-of-teachingA review-on-principles-of-teaching
A review-on-principles-of-teaching
 
Life Of Pi Art
Life Of Pi ArtLife Of Pi Art
Life Of Pi Art
 
Mksap pulmonary qa 1
Mksap pulmonary qa 1Mksap pulmonary qa 1
Mksap pulmonary qa 1
 
Science k-12-curriculum-guides
Science k-12-curriculum-guidesScience k-12-curriculum-guides
Science k-12-curriculum-guides
 
K to-12-mathematic-curriculum-guide-grade-1
K to-12-mathematic-curriculum-guide-grade-1K to-12-mathematic-curriculum-guide-grade-1
K to-12-mathematic-curriculum-guide-grade-1
 
English curriculum guide grades 1 10 december 2013
English curriculum guide grades 1 10 december 2013English curriculum guide grades 1 10 december 2013
English curriculum guide grades 1 10 december 2013
 
K to 12 Mathematics Curriculum Guide for Grades 1 to 10
K to 12 Mathematics Curriculum Guide for Grades 1 to 10K to 12 Mathematics Curriculum Guide for Grades 1 to 10
K to 12 Mathematics Curriculum Guide for Grades 1 to 10
 

Similar to Review questions-no-answer-key

Word Builders and Word Association
Word Builders and Word AssociationWord Builders and Word Association
Word Builders and Word AssociationHiacynt Perez
 
Word Builders and Word Association
Word Builders and Word AssociationWord Builders and Word Association
Word Builders and Word AssociationHiacynt Perez
 
Lesson Presentation (Edtech)
Lesson Presentation (Edtech)Lesson Presentation (Edtech)
Lesson Presentation (Edtech)projectRAE
 
Reading_Lesson 2 text as connected discourse
Reading_Lesson 2 text as connected discourseReading_Lesson 2 text as connected discourse
Reading_Lesson 2 text as connected discourseTine Lachica
 
Classes of Words
Classes of WordsClasses of Words
Classes of WordsTutik SR
 
CHAPTER I.doc
CHAPTER I.docCHAPTER I.doc
CHAPTER I.docUlilAydiy
 
Idiomi, lecture 03, 12 13
Idiomi, lecture 03, 12 13Idiomi, lecture 03, 12 13
Idiomi, lecture 03, 12 13Alen Šogolj
 
The nature of reference in text and in powerpoint presentation
The nature of reference in text and in powerpoint presentationThe nature of reference in text and in powerpoint presentation
The nature of reference in text and in powerpoint presentationRafaqat Hussain Rafaqat
 
Questions On The Stopping Of A Child Tv Channel
Questions On The Stopping Of A Child Tv ChannelQuestions On The Stopping Of A Child Tv Channel
Questions On The Stopping Of A Child Tv ChannelMia Gordon
 
grammar-for-writing-teaching-narrative.pdf
grammar-for-writing-teaching-narrative.pdfgrammar-for-writing-teaching-narrative.pdf
grammar-for-writing-teaching-narrative.pdfScarlet Rojas
 
Terminolog yand vocabhandout
Terminolog yand vocabhandoutTerminolog yand vocabhandout
Terminolog yand vocabhandoutGiovanni Tovar
 
Reading vocabulary
Reading vocabularyReading vocabulary
Reading vocabularyglenda75
 
Morphological typology/ Morphological Operations
Morphological typology/ Morphological OperationsMorphological typology/ Morphological Operations
Morphological typology/ Morphological OperationsDr. Mohsin Khan
 

Similar to Review questions-no-answer-key (20)

Word Builders and Word Association
Word Builders and Word AssociationWord Builders and Word Association
Word Builders and Word Association
 
Word Builders and Word Association
Word Builders and Word AssociationWord Builders and Word Association
Word Builders and Word Association
 
Lesson Presentation (Edtech)
Lesson Presentation (Edtech)Lesson Presentation (Edtech)
Lesson Presentation (Edtech)
 
DLL_ENGLISH 6_Q1_W1.docx
DLL_ENGLISH 6_Q1_W1.docxDLL_ENGLISH 6_Q1_W1.docx
DLL_ENGLISH 6_Q1_W1.docx
 
CONTEXT CLUES.pptx
CONTEXT CLUES.pptxCONTEXT CLUES.pptx
CONTEXT CLUES.pptx
 
Syntax.ppt
Syntax.pptSyntax.ppt
Syntax.ppt
 
Reading_Lesson 2 text as connected discourse
Reading_Lesson 2 text as connected discourseReading_Lesson 2 text as connected discourse
Reading_Lesson 2 text as connected discourse
 
Classes of Words
Classes of WordsClasses of Words
Classes of Words
 
DLL_ENGLISH 6_Q1_W1.docx
DLL_ENGLISH 6_Q1_W1.docxDLL_ENGLISH 6_Q1_W1.docx
DLL_ENGLISH 6_Q1_W1.docx
 
CHAPTER I.doc
CHAPTER I.docCHAPTER I.doc
CHAPTER I.doc
 
Idiomi, lecture 03, 12 13
Idiomi, lecture 03, 12 13Idiomi, lecture 03, 12 13
Idiomi, lecture 03, 12 13
 
导论
导论导论
导论
 
语法导论
语法导论语法导论
语法导论
 
Grammar Session.pptx
Grammar Session.pptxGrammar Session.pptx
Grammar Session.pptx
 
The nature of reference in text and in powerpoint presentation
The nature of reference in text and in powerpoint presentationThe nature of reference in text and in powerpoint presentation
The nature of reference in text and in powerpoint presentation
 
Questions On The Stopping Of A Child Tv Channel
Questions On The Stopping Of A Child Tv ChannelQuestions On The Stopping Of A Child Tv Channel
Questions On The Stopping Of A Child Tv Channel
 
grammar-for-writing-teaching-narrative.pdf
grammar-for-writing-teaching-narrative.pdfgrammar-for-writing-teaching-narrative.pdf
grammar-for-writing-teaching-narrative.pdf
 
Terminolog yand vocabhandout
Terminolog yand vocabhandoutTerminolog yand vocabhandout
Terminolog yand vocabhandout
 
Reading vocabulary
Reading vocabularyReading vocabulary
Reading vocabulary
 
Morphological typology/ Morphological Operations
Morphological typology/ Morphological OperationsMorphological typology/ Morphological Operations
Morphological typology/ Morphological Operations
 

Recently uploaded

NOTES OF DRUGS ACTING ON NERVOUS SYSTEM .pdf
NOTES OF DRUGS ACTING ON NERVOUS SYSTEM .pdfNOTES OF DRUGS ACTING ON NERVOUS SYSTEM .pdf
NOTES OF DRUGS ACTING ON NERVOUS SYSTEM .pdfSumit Tiwari
 
HED Office Sohayok Exam Question Solution 2023.pdf
HED Office Sohayok Exam Question Solution 2023.pdfHED Office Sohayok Exam Question Solution 2023.pdf
HED Office Sohayok Exam Question Solution 2023.pdfMohonDas
 
How to Print Employee Resume in the Odoo 17
How to Print Employee Resume in the Odoo 17How to Print Employee Resume in the Odoo 17
How to Print Employee Resume in the Odoo 17Celine George
 
How to Make a Field read-only in Odoo 17
How to Make a Field read-only in Odoo 17How to Make a Field read-only in Odoo 17
How to Make a Field read-only in Odoo 17Celine George
 
3.19.24 Urban Uprisings and the Chicago Freedom Movement.pptx
3.19.24 Urban Uprisings and the Chicago Freedom Movement.pptx3.19.24 Urban Uprisings and the Chicago Freedom Movement.pptx
3.19.24 Urban Uprisings and the Chicago Freedom Movement.pptxmary850239
 
The Singapore Teaching Practice document
The Singapore Teaching Practice documentThe Singapore Teaching Practice document
The Singapore Teaching Practice documentXsasf Sfdfasd
 
Patient Counselling. Definition of patient counseling; steps involved in pati...
Patient Counselling. Definition of patient counseling; steps involved in pati...Patient Counselling. Definition of patient counseling; steps involved in pati...
Patient Counselling. Definition of patient counseling; steps involved in pati...raviapr7
 
CapTechU Doctoral Presentation -March 2024 slides.pptx
CapTechU Doctoral Presentation -March 2024 slides.pptxCapTechU Doctoral Presentation -March 2024 slides.pptx
CapTechU Doctoral Presentation -March 2024 slides.pptxCapitolTechU
 
Patterns of Written Texts Across Disciplines.pptx
Patterns of Written Texts Across Disciplines.pptxPatterns of Written Texts Across Disciplines.pptx
Patterns of Written Texts Across Disciplines.pptxMYDA ANGELICA SUAN
 
How to Show Error_Warning Messages in Odoo 17
How to Show Error_Warning Messages in Odoo 17How to Show Error_Warning Messages in Odoo 17
How to Show Error_Warning Messages in Odoo 17Celine George
 
The Stolen Bacillus by Herbert George Wells
The Stolen Bacillus by Herbert George WellsThe Stolen Bacillus by Herbert George Wells
The Stolen Bacillus by Herbert George WellsEugene Lysak
 
How to Add a New Field in Existing Kanban View in Odoo 17
How to Add a New Field in Existing Kanban View in Odoo 17How to Add a New Field in Existing Kanban View in Odoo 17
How to Add a New Field in Existing Kanban View in Odoo 17Celine George
 
Diploma in Nursing Admission Test Question Solution 2023.pdf
Diploma in Nursing Admission Test Question Solution 2023.pdfDiploma in Nursing Admission Test Question Solution 2023.pdf
Diploma in Nursing Admission Test Question Solution 2023.pdfMohonDas
 
Education and training program in the hospital APR.pptx
Education and training program in the hospital APR.pptxEducation and training program in the hospital APR.pptx
Education and training program in the hospital APR.pptxraviapr7
 
How to Add a many2many Relational Field in Odoo 17
How to Add a many2many Relational Field in Odoo 17How to Add a many2many Relational Field in Odoo 17
How to Add a many2many Relational Field in Odoo 17Celine George
 
Ultra structure and life cycle of Plasmodium.pptx
Ultra structure and life cycle of Plasmodium.pptxUltra structure and life cycle of Plasmodium.pptx
Ultra structure and life cycle of Plasmodium.pptxDr. Asif Anas
 
How to Add Existing Field in One2Many Tree View in Odoo 17
How to Add Existing Field in One2Many Tree View in Odoo 17How to Add Existing Field in One2Many Tree View in Odoo 17
How to Add Existing Field in One2Many Tree View in Odoo 17Celine George
 
3.21.24 The Origins of Black Power.pptx
3.21.24  The Origins of Black Power.pptx3.21.24  The Origins of Black Power.pptx
3.21.24 The Origins of Black Power.pptxmary850239
 
Latin American Revolutions, c. 1789-1830
Latin American Revolutions, c. 1789-1830Latin American Revolutions, c. 1789-1830
Latin American Revolutions, c. 1789-1830Dave Phillips
 

Recently uploaded (20)

NOTES OF DRUGS ACTING ON NERVOUS SYSTEM .pdf
NOTES OF DRUGS ACTING ON NERVOUS SYSTEM .pdfNOTES OF DRUGS ACTING ON NERVOUS SYSTEM .pdf
NOTES OF DRUGS ACTING ON NERVOUS SYSTEM .pdf
 
HED Office Sohayok Exam Question Solution 2023.pdf
HED Office Sohayok Exam Question Solution 2023.pdfHED Office Sohayok Exam Question Solution 2023.pdf
HED Office Sohayok Exam Question Solution 2023.pdf
 
Prelims of Kant get Marx 2.0: a general politics quiz
Prelims of Kant get Marx 2.0: a general politics quizPrelims of Kant get Marx 2.0: a general politics quiz
Prelims of Kant get Marx 2.0: a general politics quiz
 
How to Print Employee Resume in the Odoo 17
How to Print Employee Resume in the Odoo 17How to Print Employee Resume in the Odoo 17
How to Print Employee Resume in the Odoo 17
 
How to Make a Field read-only in Odoo 17
How to Make a Field read-only in Odoo 17How to Make a Field read-only in Odoo 17
How to Make a Field read-only in Odoo 17
 
3.19.24 Urban Uprisings and the Chicago Freedom Movement.pptx
3.19.24 Urban Uprisings and the Chicago Freedom Movement.pptx3.19.24 Urban Uprisings and the Chicago Freedom Movement.pptx
3.19.24 Urban Uprisings and the Chicago Freedom Movement.pptx
 
The Singapore Teaching Practice document
The Singapore Teaching Practice documentThe Singapore Teaching Practice document
The Singapore Teaching Practice document
 
Patient Counselling. Definition of patient counseling; steps involved in pati...
Patient Counselling. Definition of patient counseling; steps involved in pati...Patient Counselling. Definition of patient counseling; steps involved in pati...
Patient Counselling. Definition of patient counseling; steps involved in pati...
 
CapTechU Doctoral Presentation -March 2024 slides.pptx
CapTechU Doctoral Presentation -March 2024 slides.pptxCapTechU Doctoral Presentation -March 2024 slides.pptx
CapTechU Doctoral Presentation -March 2024 slides.pptx
 
Patterns of Written Texts Across Disciplines.pptx
Patterns of Written Texts Across Disciplines.pptxPatterns of Written Texts Across Disciplines.pptx
Patterns of Written Texts Across Disciplines.pptx
 
How to Show Error_Warning Messages in Odoo 17
How to Show Error_Warning Messages in Odoo 17How to Show Error_Warning Messages in Odoo 17
How to Show Error_Warning Messages in Odoo 17
 
The Stolen Bacillus by Herbert George Wells
The Stolen Bacillus by Herbert George WellsThe Stolen Bacillus by Herbert George Wells
The Stolen Bacillus by Herbert George Wells
 
How to Add a New Field in Existing Kanban View in Odoo 17
How to Add a New Field in Existing Kanban View in Odoo 17How to Add a New Field in Existing Kanban View in Odoo 17
How to Add a New Field in Existing Kanban View in Odoo 17
 
Diploma in Nursing Admission Test Question Solution 2023.pdf
Diploma in Nursing Admission Test Question Solution 2023.pdfDiploma in Nursing Admission Test Question Solution 2023.pdf
Diploma in Nursing Admission Test Question Solution 2023.pdf
 
Education and training program in the hospital APR.pptx
Education and training program in the hospital APR.pptxEducation and training program in the hospital APR.pptx
Education and training program in the hospital APR.pptx
 
How to Add a many2many Relational Field in Odoo 17
How to Add a many2many Relational Field in Odoo 17How to Add a many2many Relational Field in Odoo 17
How to Add a many2many Relational Field in Odoo 17
 
Ultra structure and life cycle of Plasmodium.pptx
Ultra structure and life cycle of Plasmodium.pptxUltra structure and life cycle of Plasmodium.pptx
Ultra structure and life cycle of Plasmodium.pptx
 
How to Add Existing Field in One2Many Tree View in Odoo 17
How to Add Existing Field in One2Many Tree View in Odoo 17How to Add Existing Field in One2Many Tree View in Odoo 17
How to Add Existing Field in One2Many Tree View in Odoo 17
 
3.21.24 The Origins of Black Power.pptx
3.21.24  The Origins of Black Power.pptx3.21.24  The Origins of Black Power.pptx
3.21.24 The Origins of Black Power.pptx
 
Latin American Revolutions, c. 1789-1830
Latin American Revolutions, c. 1789-1830Latin American Revolutions, c. 1789-1830
Latin American Revolutions, c. 1789-1830
 

Review questions-no-answer-key

  • 1. 1 LICENSURE EXAMINATIONS FOR TEACHERS (LET) Reviewer for ENGLISH Competencies:  Arrive at meanings of words using context clues, structural analysis and other word formation techniques  Note details to discover the central theme of a passage  Point out the organization structure of a passage and determine how the parts are related to the whole  Draw inferences and implications on reading texts  Distinguish the different parts of speech in English – their meanings, forms, order, and functions  Analyze sentences in terms of their constituents elements  Identify the rules of grammatical usage Prepared by: Ruth A. Alido Philippine Normal University, Manila VOCABULARY Vocabulary knowledge implies a rich understanding of the word. It means knowing a word by definition and associating experiences with that word. 1. Structural Analysis. Words are made up of the smallest meaningful units called morphemes. The visual scrutiny of unfamiliar words to identify morphemes is called structural analysis. Knowing the roots, prefixes, and suffixes of words helps reveal the meaning of the total word form. a) Root words are words from which other words are formed by adding a beginning part (prefix) or an ending part (suffix) active (move) porter (carry) contradiction (to speak) b) Prefixes are word parts added to the beginning of a word benevolent (good) decline (from) nonsense (not) c) Suffixes are syllables added at the end of a word to form a new word with a different meaning Biology (study of) homeless (without) scientist (one who does) 2. Word Formation. Words undergo changes. The following are five processes of word formation: a) Clipping means to cut off the beginning or the end of the word. It may mean cutting from both ends, leaving a part to stand for the whole. phone photo psycho trigo chem. b) Blending is formed by fusing or putting two words together. Usually the first part of one word is blended or fused with the last part of another. The blended word then gets its meaning from the two words put together. Eurasian Philhealth cosmonaut smog telecast c) Compounding uses two or three words put together to make a full form. Most often the meaning of the word is different from its parts. Sometimes it is the meaning of the two words put together. tightwad blackout first aid runner-up trigger-happy d) Acronymy is the use of initial letter or syllables of several words in succession. PPSTA UNESCO AWOL scuba radar e) Folk or Popular Etymology results from changing a word in part or in whole to make it more like a familiar word. belfry isle bachelor barbeque caesarian
  • 2. 2 3. Context Clues. The meaning of a word may be determined by its environment – the words that surround it, either coming before or after it in the sentence of in the paragraph. a) Definition is considered the simplest and most obvious way by which the meaning of a word is revealed. The be verb is used to equate the term to be defined to the familiar word in the sentence. Psychiatry is the branch of medicine that deals with the diagnosis, treatment, and prevention of mental disorders. b) Restatement may be in the form of synonyms, examples, elaboration by the use of modifiers, and pairing closely related words. It is usually introduced by signal words: that is, for example, such as, like, in the way, that, in other words, what this means, etc. It may also be signaled by the dash and the parentheses. He was a true peripatetic, like the walking philosophers who followed Aristotle and the wandering Jews of Israel. c) Synonym is a word that means essentially the same thing as another word. It is usually preceded by the function word or. Sometimes the synonym may be found in another sentence within the paragraph. God is omniscient, or all-knowing. d) Antonym is the opposite or contrasting word that may serve as a clue to the unfamiliar word. Julia’s remarks are generally not nebulous, but clear. e) Inference requires reading between lines to see connections and relationships not explicitly stated in a particular passage. The practicing physician as a rule is completely unskilled in obstetrics. His preliminary training contains little or nothing concerning the details of pregnancy and childbirth. f) Homophones & Words with Multiple Meanings indicate several meanings that a word has in English. Susan bought a bar of soap. The soldiers tried to bar the enemies. The brilliant student passed the bar examination. g) Words of Various Disciplines are the basic meaning of specialized vocabulary in a particular subject area. The capital of the Philippines is Metro Manila. (Social Science) His capital for the new business is one million pesos. (Math) h) Multiple Context make the meaning of unfamiliar words clearer and easily understood. Butterflies fly from flower to flower. How high did the boy fly his kite? Airplanes fly regularly from Manila to Hongkong. 4. Idioms. An idiom is an expression peculiar to a language. It usually has several associated meanings which are not readily understandable from its grammatical construction and cannot be derived from the meaning of its separate elements.  cross to bear – endure a kind of burden  close-fisted – a miser  cold-reception – unfriendly  broad daylight – open, full daylight  blackmail – money extorted by threat of intimidation  Argus-eyed – jealously watchful  what not -etcetera  blue-blood – noble blood  chicken-hearted – a timid, cowardly fellow  absent-minded – to be inattentive of what is going on  fair weather friend – a friend who deserts you in time of difficulties  henpecked husband – a submissive husband  laughing stock – object of ridicule  greenhorn – an inexperienced person  by heart – from memory  burn the midnight oil – study or work far into the night  bundle from heaven – new-born baby  belle of the ball – most popular girl  blanket authority – complete right or privilege  behind the times – unprogressive  came to terms – deal with misunderstanding
  • 3. 3 5. Figures of Speech. These forms of expressions are used to convey meaning or heighten effect, often by comparing or identifying one thing with another that has meaning or connotation familiar to the reader or listener. a) Simile – a comparison between two unlike objects by using like or as His mind is like a sponge. b) Metaphor – an indirect comparison of unlike objects She is a phantom of delight c) Personification – the giving of human characteristics & capabilities to nonhuman things such as inanimate objects, abstract ideas, or animals The clouds cried a torrent of tears. d) Apostrophe – an address to the absent as if present or to the inanimate as if human “O wind, if winter comes, can spring be far behind?” e) Hyperbole – the use of excessive exaggeration for effect Waves mountain high broke over the reef. f) Oxymoron – the combining of contraries to portray a particular image or to produce a striking effect Parting is such sweet sorrow. g) Paradox – uses a phrase or statement that on surface seems contradictory, but makes some kind of emotional sense Let us go to war for peace. h) Metonymy – substitutes a word that closely relates to a person or thing Have you no respect for gray hairs? i) Synecdoche – uses a part to represent the whole No busy hand provoke a tear/No roving foot shall crush thee here j) Litotes – makes a deliberate understatement used to affirm by negating its opposite Regine Velasquez is not a bad singer. k) Irony – the opposite of what is expected is what happens To cry like a baby, that’s a fine way for a man to act. l) Allusion – refers to a literary, biblical, historical, mythological, scientific event, character, or place Beware of the kiss of Judas! m) Antithesis – a contrast of words or ideas She looks like an innocent flower but watch out for the serpent under it 6. Rhetorical Devices. These are sound devices used to convey meaning through rhyme and rhythm. a) Onomatopoeia – uses a word having a sound that imitates what it denotes hiss, bang, buzz, hush , swoosh b) Alliteration – involves the repetition of initial consonant sounds wicked and wan, threatening throngs c) Assonance – uses repetition of vowels without repetition of consonants, also called a vowel rhyme alone, alone, all, all, alone d) Consonance – repeats the final consonant sounds, also called a slant rhyme dreary and weary; odds and ends e) Rhyme – employs identical sounds from the vowel of the accented syllables to the end hold, told, mold, gold; die, sky, my, fly, pie f) Anaphora – repeats a word or words at the beginning of two or more successive clauses or verses Cannons to the right of them/Cannons to the left of them V OCABULARY PRACTICE A. Give the meaning of the underlined word based on the meaning of its root given in the parenthesis after each sentence. 1. A portable (to carry) video CD is very convenient. _________________________. 2. The people admire their leader for his tenacious (to hold) stand. ______________________. 3. The testimony of the witness was credible (to believe). _________________________. 4. The old man’s faith guided him in the tortuous (to twist) path of life. __________________. 5. The old theater had to be renovated (new). _________________________. 6. His unpleasant remarks caused a rupture (to break) in their friendship. _________________. 7. The spectators (to see) were amused by the magician’s clever antics. ___________________.
  • 4. 4 8. The aid that the victim received from civic spirited citizens was superfluous (to flow). _____. 9. The crowd became bored listening to the loquacious (to speak) politicians. ______________. 10. Syphilis is a malignant (evil) disease. _________________________. B. Pick out the meaning of the underlined word in the sentence following the meaning of the prefix. 1. A bilingual (two) dictionary is very helpful to students of philology. A. many languages C. a single language B. two languages D. the second language 2. After many years of foreign rule, the Philippines enjoyed autonomy (self). A. self-government C. freedom of speech B. peace and prosperity D. self-expression 3. The body was exhumed (out of) for another autopsy. A. buried B. consecrated C. dug out D. hung out 4. Let this fine gesture be the precedence (before). A. preparation B. priority in time C. example of prudence D. presentation 5. The panorama (whole) in Batanes is breathtakingly beautiful. A. entire view B. live music C. landscape D. natural resource 6. King George abdicated (away from) in favor of his brother. A. objected to his brother C. denounced his throne B. renounced his throne D. abducted his brother 7. The comedian used many hyperboles (over) to make his audience laugh. A. allusions B. comparisons C. exaggerations D. contrasts 8. Patients suffering tuberculosis were segregated (apart). A. placed with others C. refused admission B. placed in pay wards D. separated from others 9. Worry and fatigue can befog (completely) our thinking. A. obscure B. clarify C. becalm D. bedeck 10. A person with that personality cannot remain a non-entity (not) for long. A. one who lacks money C. one of no importance B. one of no power D. one without identity C. Choose the correct suffix for the word that best fit the blank. 1. The physician was conducting a __________ research. A. scientism B. scientific C. scientist 2. The liquor is strong. It has high __________. A. potential B. potentate C. potency 3. The body cannot be identified. It is at the state of __________. A. decomposition B. decomposable C. decomposer 4. Mr. Flavier worked in favor of population control. He is an __________ of slavery. A. advocacy B. advocate C. advocation 5. After a year’s stay abroad, the scholar was glad to turn __________. A. homely B. homeless C. homeward 6. The general manager gave Ms. Santos a duty to perform. Ms. Santos was the __________. A. employer B. employee C. employment 7. The criminal was very sorry for the crime he committed. He was __________. A. repenter B. repentant C. repentance 8. The doctor announced that his sickness was not serious it was __________. A. remedy B. remedial C. remediable 9. The professor used terms that were too general. His students wished he would be more ____. A. specific B. specify C. specification 10. The senator is engaged in the production, distribution, and consumption of wealth and of the means of supplying the material needs of his constituents. He is a well-known __________. A. economist B. economics C. economy D. Choose the correct meaning of the underlined idiom in the sentence. 1. The wife shed crocodile tears when her cruel mother-in-law died. A. sincere grief B. pretended sorrow C. continuous flow of tears 2. When the foreigner first stepped on the Philippine soil, he was all at sea about everything. A. confused B. at fault C. disappointed
  • 5. 5 3. The newly married couple executed a French leave before the reception was over. A. an aristocratic way of saying good-bye B. leaving without giving due notice of one’s departure C. waving one’s hands during departure 4. The daughter bares her soul to her mother. A. confides with some reservation B. opens for inspection C. confesses innermost thoughts 5. From the sixth floor of the building, we got a bird’s eye view of the city. A. view from a telescope B. view over the entire area C. photograph of a bird 6. They are trying to find out what is really at the bottom of the continuous rise in the prices of commodities. A. purpose B. reason C. procedure employed 7. His parent’s hopes turned to ashes in the mouth when he married soon after graduation. A. overwhelming joy B. burned to ashes C. bitter disappointment 8. The enemies entered the city armed to the teeth. A. heavily armed B. with covered mouths C. with weapons between the teeth 9. The audience was all ears to the distinguished orator. A. covering both ears B. making a compliment C. listening very attentively 10. The man is very angry. Don’t speak a single word otherwise you will be adding fuel to the fire. A. provoking further strife B. making his illness worse C. adding firewood to make the fire alive E. Identify the figures of speech and rhetorical devices used in the sentences below. 1. The dragonfly huge like a blue thread loomed from the sky. 2. Sometimes too hot the eye of heaven shines. 3. She sells seashells on the seashore. 4. Each flower has wept and bowed toward the east. 5. Ten thousand daffodils saw I at a glance. 6. Mighty Hades, why, they power hovers! 7. I did not realize I had a Hamlet for a husband. 8. There is emptiness in wholeness. 9. Her speech is a newly sharpened scythe. 10. Address that question to the chair. 11. The poor need roofs over their heads. 12. The night has a thousand eyes. 13. Oh, Liberty! How many crimes are committed in thy name! 14. The United Nations Forum is a tower of Babel. 15. Sad storms whose tears are vain. READING COMPREHENSION Reading comprehension questions usually fall into several general categories. 1. Main Idea. This usually refers to the passage as a whole, not to some segment or part of the passage. Questions are usually about the main idea or theme of the passage, about a possible title, or about the author’s primary objective. The main idea is typically (but not always) found in the first paragraph. It is the statement that gives the overall theme of the passage. In many cases, it is in the form of an argument, including a premise and conclusion.
  • 6. 6 World War II brought new demands and needs to the nation in the 1940s. Financing the war meant additional taxes and changes to payroll processing. An armed force with millions of people produced new challenges in administration and record keeping. New weapons required countless calculations and tests. Defense research demanded the development of large-scale computing devices. 2. Supporting Ideas. This is about the idea expressed in one part of the passage rather than about the passage as a whole. This type of question is to distinguish between the main idea and those themes that support it, some of which may be implicit or implied rather than explicitly stated. Males and females are each associated with different kinds of behavior, and what is considered masculine and feminine differs from society to society. These concepts of masculinity and femininity extend to how people walk, sit, talk, and dress. In our society, as in all others, men walk and talk in certain ways and until very recently dressed very differently from women. In some societies, different spatial areas are associated with males and females – women in many Middle Eastern societies are restricted to certain parts of the house and may only come into contact with the males who are members of their family. In such societies, the coffee house and the market are defined as male domains. In contrast, in some African societies, women predominate in the marketplace. 3. Drawing Inferences. This asks about ideas that are not explicitly stated in a passage. The question refers meanings implied by the author based on information given in the passage. The procedure is actually quite simple. First, you arrange the items into different groups. Of course one pile may be sufficient depending on how much there is to do. If you have to go somewhere else due to lack of facilities that is the next step; otherwise, you are pretty well set. It is important not to overdo things. That is, it is better to do too few things at once than too many. In the short run this may not seem important but complications can easily arise. A mistake can be expensive as well. At first, the whole procedure will seem complicated. Soon, however, it will become just another facet of life. It is difficult to foresee any end to the necessity for this task in the immediate future, but then, one never can tell. After the procedure is completed one arranges the materials into different groups again. Then they can be put into their appropriate places. Eventually they will be used once more and the whole cycle will then have to be repeated. However, that is part of life. 4. Specific Details. This type of questions asks about specific facts or details the author has stated explicitly in the passage. Today, farmers grow about 449 million tons of corn worldwide. In terms of area under cultivation, it’s the world’s second or third largest crop. Providing 19 percent of the world’s food calories and 15 percent of its food-crop protein, global corn production yields about 200 pounds of the cereal for every individual alive. It should come as no surprise, then, that corn – also known as maize – is a staple for some 200 million people, including nearly half the world’s chronically malnourished. However, conventional corn is deficient in the vitamin niacin, and roughly half its protein in lacks lysine and tryptophan – two essential amino acids. As a result, this staple is not a sufficient protein source, for children, the sick, and pregnant or lactating women. 5. Tone or Attitude of the Passage. This concentrates on the author’s style, attitude, or mood. The use of key words such as adjectives that reveal if the author is “pessimistic,” “critical,” “supportive,” or “objective” about an event, idea, or situation in the passage help determine the tone or attitude. Them dirty lousy politicians is getting altogether too high and might, the way they is always arranging to take advantage of the little businessmen by raising up the tax payments and collecting more money from the little fellows. They ain’t nothing much can be done about this here business, because them politicians has certainly got the inside connections and they always work through undercover arrangements. It’s might funny that the generals and the admirals and the presidents of the big corporations aren’t paying out no oversized tax installments but just the little businessmen who aren’t getting much money nohow.
  • 7. 7 6. The Logical Structure of the Passage. This tests the overall meaning, logic, or organization of a passage. The question asks how several ideas in a passage are interrelated or how a passage is constructed, classifies, compares, describes events, or situations. Let’s begin then to investigate the characteristics of your memory system as it now operates. You may be surprised to discover that there is already more to your memory system than you realize even existed. Scientific investigations of memory and how it works have turned up the fact that each of us actually has three completely different types of memory. These memory systems are called immediate memory, short- term memory, and long-term memory. Each of these retains, and loses, information differently. In addition, the life span of information within each system varies. Consequently, each is use for different purposes and we must learn how to use each most effectively. Whenever something is to be remembered for only a short period, it can go into short-term memory; it if is to be used immediately, immediate memory is where it belongs. 7. Determining the Meaning of Words from the Context. When a question asks for the meaning of a word, it can usually e deduced from the context of the passage. Nature loves edges. Take a place where mountain meets plain, field joins forest, or river fuses with ocean. Abundant wildness usually occurs where one kind of habitat hits another. At a good edge the number and variety of living things – the quotient of ecological diversity – generally exceed what can be found in a more uniform habitat. And because humans appreciate visual contrasts, edges often mean aesthetic distinctness too. So it is with an estuary, the biological hot spot and scenic climax where freshwater and saltwater first meet head on. PRACTICE on BASIC READING SKILLS 1. Skimming and Scanning. Skimming is the selective reading of material to survey the purpose and the general idea. Scanning, on the other hand, is a quick search for a specific information in the text. Read the selection below and answer the questions that follow. The text is written in such a way to practice skimming and scanning. Today’s English has many words…early myths. Some…changed…not recognizable…mythological. Others remain…original form. Greeks believe…beginning…things…existed…disorder…empty space… condition…space…called chaos…today means disorder…complete disorganization… described…chaotic. …chaos…conceived…ordered world called ge…origin…source…things. Today ge…common part…English words…related…earth, i.e., geology, geography, geometry. …Greek god…sky was Uranus. Today we have…planet…Uranus…chemical element uranium…rain…came…sky fertilized earth…union came… Titans,… race… giants…great strength…anything…great size…strength…called titanic. One…titans was Saturn. One of the planets…called Saturn…day…Saturday named…him…word saturnine describes…gloomy person. …Titans,…new generation…gods came…father was Jove…adjective, jovial means ‘cheerful.’ Jove lived…heavens…brother Hades, god…dead, lived below… sister. Ceres, was…goddess…agriculture…cereal…named…her. 1. The word chaotic A. means ‘erotic’ C. means ‘something disorganized’ B. is the same as chaos D. is a myth 2. The Greeks thought of the word ge which means A. earth C. origin of all things B. heaven D. rain 3. The Titan god of the sky was A. Uranus B. Ceres C. Jove D. Hades
  • 8. 8 4. The statement that best approximates the main idea is A. English is a combination of many languages. B. Many words from Greek mythology have undergone little change in meaning. C. The myths of early culture have enriched language. D. English contains many words from Greek mythology which have altered in meaning and in form. 5. A good title for this selection is A. The Mystery of the English Language B. The Influence of the Past on Language C. Classical Mythology in Everyday English D. The Varied Meanings of Words. 2. Finding the Topic Sentence. The topic sentence gives the key to an entire paragraph. It is usually found in the beginning of a paragraph. However, there is no absolute rule. Some paragraphs build up to a conclusion hence the key sentence is at the end. Identify the topic sentence in each of the following examples. Example 1. The world faces a serious problem of overpopulation. Right now many people starve from lack of adequate food. Efforts are being made to increase the rate of food production, but the number of people to be fed increases at a faster rate. Topic Sentence: _______________________________________________________ Example 2. During the later years of the American Revolution, the Articles of Confederation government was formed. This government suffered severely from a lack of power. Each state distrusted the others and gave little authority to the central or federal government. The Articles of Confederation produced a government that could not raise money from taxes, prevent Indian raids, or force the British out of the United States. Topic Sentence: _______________________________________________________ Example 3. They had fewer men available as soldiers. Less than one-third of the railroads and only a small proportion of the nation’s industrial production was theirs. For most of the war their coastline was blockaded by Northern ships. It is a tribute to Southern leadership and the courage of the people that they were not defeated for four years. Topic Sentence: _______________________________________________________ 3. Finding the General Theme. A more advanced skill is the ability to read several paragraphs and relate them to one general theme or main idea. This involves careful reading of the entire passage and deciding which idea is the central or main one. The general theme is usually most frequent or most important and every sentence relates to it. In order to find the general theme: a. Read at normal speed. b. Locate the topic sentence in each paragraph. c. Note ideas that are frequent or emphasized. d. Find the idea to which most of the passage is related. Identify the general theme in the example below. Note the underlined parts as you read the passage. True democracy means direct rule by the people. A good example can be found in a modern town meeting in many small New England towns. All citizens aged twenty-one or over may vote. They not only vote for officials, but they also get together to vote on local laws (or ordinances). The small size of the town and the limited number of voters make this possible. In the cities, voters cast ballots for officials who get together to make the laws. Because the voters do not make the laws directly, this system is called indirect democracy or representative government. There is no problem of distance to travel, but it is difficult to run a meeting the hundreds of thousands of citizens. Representation of voters and direct voice in making laws are more of a problem in state or national governments. The numbers of citizens and the distances to travel make representative government the most practical way to make laws.
  • 9. 9 The main theme of the passage is that A. the United States is not democratic B. representative government does not make laws C. every citizen makes laws directly D. increasing populations lead to less direct democracy 4. Finding Logical Relationships. The best way to fully understand the meaning of a passage is to look for the general theme and then relate the ideas and opinions found in the passage to this general theme. It is important to determine not only what is important in the passage but also how the ideas interrelate to form the whole. As you read the passage, look for general theme and supporting facts, words or phrases that signal emphasis or shift in thought, and the relation of one idea to another. The candidate who wants to be elected pays close attention to statements and actions that will make the voters see him favorably. In ancient Rome candidates wore pure white togas (the Latin word candidates means “clothed in white”) to indicate that they were pure, clean, and above any “dirty work.” However, it is interesting to note that such a toga was not worn after election. In more modern history, candidates have allied themselves with political parties. Once a voter knows and favors the views of a certain political party, he may vote for anyone with that party’s label. Nevertheless, divisions of opinion develop, so that today there is a wide range of candidate views in any major party. The best conclusion to be drawn from the first paragraph is that after an election A. all candidates are dishonest B. candidates are less concerned with symbols of integrity C. candidates do not change their ideas D. policies always change 5. Making Inferences. An inference is not stated. It is assumed by the reader from something said by the writer. An inference is the likely or probable conclusion rather than the direct, logical one. It usually involves an opinion or viewpoint that the writer wants the reader to follow or assume. Look for clues that reveal the writer’s viewpoint in the passage below. Example. Slowly but surely the great passenger trains of the United States have been fading from the rails. Short-run commuter trains still rattle in and out of the cities. Between major cities you can still find a train, but he schedules are becoming less frequent. The Twentieth Century Limited, The Broadway Limited, and other luxury trains that sang along the rails at 60 to 80 miles an hour are no longer running. Passengers on other long runs complain of poor service, old equipment, and costs in time and money. The long distance traveler today accepts the noise of jets, the congestion at airports, and the traffic between airport and city. A more elegant and graceful way is becoming only a memory. 1. With respect to the reduction of long-run passenger trains, this writer expresses A. regret B. pleasure C. grief D. anger 2. The author seems to feel that air travel is A. costly B. slow C. elegant D. uncomfortable READING COMPREHENSION STRATEGIES  Answer passages with familiar subject matter first.  Read the questions first, then the passage.  Underline parts of the passage.  Read all the answer alternatives.  Learn to identify the major question types.
  • 10. 10 GRAMMAR It is important to recognize common errors in grammar and usage based on the basic rules of grammar. A. Verb Errors 1. Verb Tense. Check if the correct verb tense has been used in the sentence.  When I came home, the children still didn’t finish dinner.  When I came home, the children still hadn’t finished dinner. In reported speech, check that the rule of sequence of tenses has been observed.  She promised she will come.  She promised she would come. 2. Tense Formation. Know the past participle of irregular verbs.  He throwed it out the window.  He threw it out the window. 3. Subject-Verb Agreement. Check if the verb agrees with the subject in number.  There is many reasons why I can’t help you.  There are many reasons why I can’t help you. 4. Conditional Sentences. The word if will NEVER be followed by the words will or would.  If I would have known, I wouldn’t have gone.  If I had known, I wouldn’t have gone. 5. Expressions of Desire. Unfulfilled desires are expressed by the form “________ had hoped that ________ would (or could, or might) do ________.”  I wish I heard that story about him before I met him.  I wish I had heard (or could have heard or would have heard) that story about him before I met him. 6. Verbs Followed by Verb Words. A verb word is the infinitive without the to.  She ignored the doctor’s recommendation that she stops smoking.  She ignored the doctor’s recommendation that she stop smoking. 7. Tag Endings. Check for three things in tag endings: a) Does the ending use the same person as the sentence verb? b) Does the ending use the same tense as the sentence verb? c) If the sentence verb is positive, is the ending negative; if the sentence verb is negative, is the ending positive?  She’s been there before, isn’t she?  She’s been there before, hasn’t she? 8. Negative Imperatives. There are two forms for negative imperatives.  Would you please don’t smoke here.  Please don’t smoke here. or Would you please not smoke here. 9. Affirmative and Negative Agreement of Verbs. There are two correct forms for both the affirmative and negative agreements.  I haven’t seen the film and hasn’t either.  I haven’t seen the film and she hasn’t either. or I haven’t seen the film and neither has she. 10. Infinitives of Gerunds in the Complement of Verbs. Some verbs may be followed by either an infinitive or a gerund. Others may require either one or the other for idiomatic reasons.  I intend learning French next semester.  I intend to learn French next semester. 11. Verbs Requiring How in the Complement. The verbs KNOW, TEACH, LEARN, and SHOW require the word how before an infinitive in the complement.  She knows to drive.  She knows how to drive.
  • 11. 11 12. Idiomatic Verbs Expressions. There are a few commonly used idiomatic verb expressions. a. must have (done) – it is a logical conclusion They’re late. They must have missed the bus. b. had better (do) – it is advisable It’s getting cold. You had better take your coat. c. used to (do) – was in the habit of doing in the past I used to smoke a pack of cigarettes a day, but I stopped. d. to be used to – to be accustomed to The noise doesn’t bother me; I’m used to studying with the radio on. e. make someone do – force someone to do My mother made me take my little sister with me to the movies. f. would rather – would prefer I would rather you didn’t speak to her. B. Pronoun Errors 1. Pronoun Subject-Object. Check if a pronoun is the SUBJECT or the OBJECT of a verb or preposition.  All of us – Fred, Jane, Alice, and me – were late.  All of us – Fred, Jane, Alice, and I – were late. 2. Who and Whom. When in doubt about the correctness of WHO/WHOM, try substituting the subject/object of a simpler pronoun to clarify the meaning.  I don’t know who Sarah meant.  I don’t know whom Sarah meant. 3. Pronoun Subject- Verb Agreement. Check if the pronoun and its verb agree in number.  Jessa is absent, but a few of the class is here.  Jessa is absent, but a few of the class are here. 4. Possessive Pronoun Agreement. Check if possessive pronouns agree in person and number.  If anyone calls, take their name.  If anyone calls, take his name. 5. Pronouns After the Verb To Be. TO BE is an intransitive verb and will always be followed by a subject pronoun.  It must have been her at the door.  It must have been she at the door. 6. Position of Relative Pronouns. A relative pronoun refers to the word preceding it. If the meaning is unclear, the pronoun is in the wrong position.  He could park right in front of the door, which was very convenient.  His being allowed to park right in front of the door was very convenient. 7. Parallelism of Impersonal Pronouns. In forms using impersonal pronouns, use either “one… one’s/his or her” or “you… your.”  One should take your duties seriously.  One should take one’s/his or her duties seriously.  or You should take your duties seriously. C. Adjective and Adverb Errors 1. Use of Adjectives and Adverbs. Check if a word modifier is an adjective or an adverb and make sure the correct form is used.  I sure wish I were rich!  I surely wish I were rich! 2. Adjectives with Verbs of Sense. Intransitive verbs are described by adjectives while transitive verbs are modified with adverbs.  She looked very well.  She looked very good!
  • 12. 12 3. Comparatives. In using adjectives of one or two syllables ending in –y, add –er. Other words of more than one syllable use more. Adverbs of one syllable add –er; longer adverbs use more.  This exercise is harder then the last one  This exercise is harder than the last one. 4. Parallel Comparisons. Check if the correct form is used in parallel comparisons.  The more you practice, you will get better.  The more you practice, the better you will get. 5. Illogical Comparatives. Check comparisons to make sure they make sense.  Texas is bigger than any state in the United States.  Texas is bigger than any other state in the United States. 6. Identical Comparisons. Something can be the same as or like something else. Do not mix up the two forms.  Your dress is the same like mine.  Your dress is like mine. or  Your dress is the same as mine. 7. Idioms Using Comparative Structures. Some idiomatic terms are formed like comparatives although they are not true comparisons.  You may have to spend so much as two hours waiting.  You may have to spend as much as two hours waiting. 8. Noun – Adjectives. When a noun is used as an adjective, treat it as an adjective. Do not pluralize or add ‘s.  You’re talking like a two-years-old child!  You’re talking like a two-year-old child! 9. Ordinal and Cardinal Numbers. Ordinal numbers (first, second, third, etc.) are preceded by the. Cardinal numbers (one, two, three, etc.) are not.  We missed first act.  We missed the first act. 10. Modifying Countable and Non-countable Nouns. If a noun can be preceded by a number, it is a countable noun; if not it is uncountable.  I was surprised by the large amount of people who came.  I was surprised by the large number of people who came. D. Errors in Usage 1. Connectors. Do not mix different forms in connecting ideas.  She speaks not only Spanish but French as well.  She speaks Spanish and French.  She speaks Spanish. She also speaks French.  She speaks Spanish and French too.  She speaks not only Spanish but also French.  She speaks both Spanish and French.  She speaks Spanish as well as French. 2. Question Word Connectors. When a question word such as when or what is used as a connector, the clause that follows is not a question. Do not use the interrogative form.  Do you know when does the movie start?  Do you know when the movie starts? 3. Because. It is incorrect to say: The reason is because… Use: The reason is that…  The reason he was rejected was because he was too young.  The reason he was rejected was that he was too young.  He was rejected because of his young age.  He was rejected because he was too young. 4. Purpose Connectors. The word so by itself means therefore. So that means in order to or in order that.  We took a cab so we would be on time.  We took a cab so that we would be on time
  • 13. 13 5. Dangling Modifiers. An introductory verbal modifier should be directly followed by the noun or pronoun that it modifies. Such a modifier will star with a gerund of participial phrase and be followed by a comma. Look for the modified noun or pronoun immediately after the comma.  Seeing that the hour was late, it was decided to postpone the committee vote.  Seeing that the hour was late, the committee decided to postpone the vote. 6. Parallel Construction. In sentences containing a series of two or more items, check to see if the same form has been used for all the items in the series. Do not mix infinitives with gerunds, adjectives with participial phrases or verbs with nouns.  The film was interesting, exciting, and it was made well.  The film was interesting, exciting, and well-made. 7. Unnecessary Modifiers. In general, the more simply an idea is stated, the better it is.  That depends on the state of the general condition of the situation.  That depends on the situation. 8. Commonly Confused Words. Be aware of the commonly misused words in English.  He was laying in bed all day yesterday.  He was lying in bed all day yesterday. 9. Misused Words and Prepositional Idioms. Take note of the prepositions in common idioms.  They came despite of the rain.  They came in spite of the rain or  They came despite the rain. GRAMMAR PRACTICE A. Mark the letter of the poorly constructed sentence in each group. 1. A. She was a fine tennis player and she did not win a gold medal. B. The refugees had neither food nor shelter for ten days. C. He blushes; therefore, he is guilty. D. He declared that he would never give up; nevertheless, he gave up. 2. A. Having no vacation you do not think about what a person could do if he had one. B. In writing a paragraph, we shall find it easier to stick to our subject if we begin with a topic sentence. C. One never know what one may do in a moment of excitement. D. He didn’t want to argue with people, but he could not ignore unfair criticisms 3. A. Nena will come tomorrow, won’t she? B. He doesn’t deserve our sympathy, does he? C. We are fortunate, aren’t we? D. The student who enjoys his work learns quickly, isn’t it? 4. A. She is as good as her sister. C They like Josefa better than I. B. I love him like a brother. D. They can’t be better than we. 5. A. I met one of the foreign student an hour ago. C. “Who is to blame?” B. One of them is guilty. D. He was one of those who came late. 6. A. When she saw the stranger fell she utters a loud shriek. B. He tore everything he took hold of. C. By this time tomorrow we shall have finished our work. D. I had reached the shed when it started to rain. 7. A. I heard the news over the radio C. Her request for transfer was granted. B. She was absent in the meeting. D. She is expected to arrive on Monday.
  • 14. 14 8. A. He fought valiantly. B. The train goes through some provinces in Central Luzon. C. All the boats headed for the cove when the tide rose hurriedly. D. She was rather pretty. 9. A. I have read the book last week C. The visitors arrived yesterday. B. Jose would have gone home by three o’clock D. The rain has stopped 10. A. The mother as well as her daughter are here again. B. The teacher, together with her students, is in the library. C. There are a woman and a child in the room. D. Either Lily or I am going to attend the party B. In each group of sentences one does not illustrate any of the following grammar points: variation from normal word order, parallel structures, subordination for emphasis, use of modifiers for vividness of expression, economy in expression, and consistent point of view. 1. Which does not observe a consistent point of view? Set 1. A. Having no money to spare, you do not know how embarrassing it is for a person at times. B. Grandmother always boasts of her well-disciplined grandchildren. C. She can never be a nurse; she passes out at the sight of blood. D. Little Mara couldn’t wait to open the surprise package she got from Uncle Bert. Set 2. A. The farmers have now realized what the government is trying to do to help them. B. I am usually patient and tolerant with obnoxious people but a person has his limits. C. Once one has agreed to do a job, he should make every effort to really do it. D. The government in its desire to uplift the welfare of its people implements various developmental programs. 2. Which does not observe economy in expression? Set 1. A. Two hundred students joined the rally. B. Last week we had occasion to be the witness of a very interesting incident. C. The private sector is now cooperating with the government. D. If I had much money, I would travel the rest of my life. Set 2. A. The people started to wave banners and to shout “Mabuhays.” B. The boys learned to speak with eloquence and to write effectively. C. He is admired by his friends and feared by his enemies. D. A fellow who wants to earn some money and to have a little adventure will find working on a cargo ship exciting. 3. Which does not use parallel structures? Set 1. A. The contest organizers gave her the option to go on an all expense-paid tour of Spain or getting a brand new car. B. Some of the things she enjoys doing are knitting, sewing and embroidering. C. It is difficult to work with people who are irresponsible and who cannot be relied upon. D. Miss Reyes expects her pupils to study diligently and to cooperate in class activities. Set 2. A. Read silently and you should keep your eyes from wandering off the page. B. She doesn’t want to see me or talk to me. C. The athletes were encouraged by the hopes of their coaches and the support of their countrymen. D. We knew he was lying but we could not prove it.
  • 15. 15 4. Which does not show variation from normal word order? Set1. A. For me there is no excuse C. She dances gracefully. B. Slowly the men came out of their hiding place. D. One of the paintings he gave to his sister. Set 2. A. That book I read when I was in the high school. B. Fortunate are the persons who register early. C. Cecile played on quietly, passionately. D. Patiently she worked out the details of the plan. 5. Which does not use modifiers for vividness of expression? Set 1. A. She has a lovely face and a beautiful figure. C. She is as sweet as a rosebud. B. The jeepney driver was a devil on the loose. D. He works like a machine. Set 2. A. The director kept things moving at breakneck speed throughout the performance. B. The following statistics give, a good idea of the effects of tobacco. C. To their surprise the damage was not so bad as they expected. D. It was the first time in my life I had seen Baguio City. 6. Which does not use subordination for emphasis? Set 1. A. Seeing that some bystanders were hit by broken bottles, the policemen quickly blew their whistles and chased the troublemakers. B. After realizing that it was useless to fight, the girls kissed and made up. C. When my father saw that I was gasping for breath, he rushed me to the hospital. D. Sonny was relatively new in school and he though he was the most popular figure in the campus. Set 2. A. To be sure that my composition was well organized, I made an outline. B. At the university, Canny became friendly with Rene, the youngest son of the Prime Minister. C. As we approached closer to the place we had a strange feeling that we would not be received cordially. D. She was famous as a child prodigy and her fame continued throughout her lefe. TIPS TO HELP YOU COPE  Read the sentence carefully.  Check for pronoun errors, if there are none, check the verbs  If you find no errors in either verbs or pronouns, look at adjectives and adverbs.  Other possible errors include the use of incorrect idioms and faulty parallelism.  Be aware of the common grammar and usage errors
  • 16. 16 ENHANCING TEST TAKING SKILLS Part I – VOCABULARY In each question, select the word or phrase that most nearly means the same as the underlined word. Circle the letter of your answer. 1. I’m afraid you can’t see him now, he’s making the rounds. A. songs sung one after the other C. a kind of dance B. a series of professional calls on hospital patients D. a knot of people 2. I was itching all over last night so I had to consult a dermatologist. A. a bone specialist C. a blood specialist B. a skin specialist D. a kidney specialist 3. Before you start with your hobby of collecting insects, why don’t you consult an entomologist? A. an insect collector C. a person who is fond of insects B. a publisher of books on insects D. a specialist in the study of insects 4. Have you ever had an intravenous injection? A. inside the vein C. near the vein B. between the veins D. across the veins 5. Manuel is so complacent he does not have a care in the world. A. satisfied B. uneasy C. excited D. happy 6. She was talking all the time and I couldn’t get a word in. A. to go out of the room C. to make a correction B. to stop someone from talking D. to have the chance to speak 7. A bilingual person is one who speaks _______ languages. A. two B. three C. four D. five 8. Vandalism or any other form of anti-social behavior should not be condoned. A. condemned B. explained C. overlooked D. permitted 9. The president sees to it that those under him are not compelled to work. He realizes that individuals who volunteer can accomplish more. A. persuaded B. lured C. forced D. asked 10. They say that distributing land to the landless is an antidote to the rise of revolutions. A. a counteraction B. a permissive action C. a related action D. a delaying action 11. Poor Manuel, his girlfriend always leaves him hanging in the balance whenever he talks of marriage. A. bewildered B. in a state of shock C. rejected D. uncertain 12. Though friends are in accord with one another most of the time, you can also expect them to be in disagreement at some other time. A. at war B. thoughtful C. in harmony D. contradicting 13. You will never be able to trace who wrote that controversial article. It bears only a pseudonym. A. an unpopular name C. an unknown name B. a fictitious name D. a dead person’s name 14. Juanito’s numerical acuity was displayed during the Math Quiz. He is really sharp when it come to figures. A. interest B. involvement C. keenness D. meticulousness
  • 17. 17 15. Ronnie is really the toughest guy in school, he always sticks his neck out to help when someone is in trouble. A. to be always present C. to act boldly despite danger B. to fight anyone D. to call for help 16. Ruby’s skepticism over her colleague’s accomplishments makes her unpopular in the campus. A. an attitude of doubt C. to indulge in doubting B. characteristic of a doubtful person D. one who doubts 17. Immediately after her almost tragic accident, she was able to get it all together and acted as though she didn’t go through one. A. to gather one’s thoughts C. to move about quickly B. to stand upright D. to collect one’s composure under pressure 18. I assure you that my dog is docile; it can be trained very easily. A. manageable B. obedient C. cooperative D. loyal 19. He’s not an effective speaker, he can’t put across his ideas. A. to make oneself understood C. to get the interest of listeners B. to deliver a speech properly D. to comprehend what is being said 20. The professor discussed with his class a review of “War and Peace.” A. a critical article or report on some literary work B. a judicial re-examination of the decision in a case C. the process of going over a subject again in study D. a formal inspection of a military or naval force Part II – READING Read each selection carefully and answer the question about it. Circle the letter of your answer. Selection 1. White potato production holds much promise as an answer to the recurring rice shortage in much of Asia. The potential of the crop as substitute for, or supplement to rice was underscored in a series of seminars on potato production conducted by foreign and local specialists in Baguio City in Benguet. The specialists noted that cereal shortage appears to occur to yearly in rice-eating countries of the Western world. This situation may be traced to the regard given by bread-eating countries to the production of supplementary food, particularly potato. In bread-eating countries, white potato is grown extensively and is eaten as a supplement to or a substitute for wheat bread as the staple food. In rice-eating countries white potato is grown only in a limited scale and is consumed as vegetable rather than as a principle source of carbohydrate. The best-selling point of potato as cereal supplement in rice consuming Asian countries is that the crop is more versatile than rice. For instance, white potato yield per hectare harvest is four to five times more than rice. With rice, average production is from five to six tons of palay while with white potato, output is from 20 to 30 tons of tubers. 1. The selection is about A. the recurring rice shortage in Asia B. the extensive white potato production in bread eating countries C. white potato as a practical substitute for rice D. the limited production of potato in rice-eating countries 2. Which of the following best summarizes the selection? A. At a meeting of foreign and local specialists, the rice-shortage in Asia was discussed. A practical solution to this problem, according to them, is substituting white potato for rice. B. Foreign and local specialists, noting the recurring rice shortage in rice-eating countries, stressed the potential of white potato as a substitute for rice. Potato harvest per hectare is more than that of rice.
  • 18. 18 C. An answer to the recurring rice-shortage problem in Asia is white potato production. Whereas in bread-eating countries, white potato is extensively grown and is a supplement to, or substitute for wheat bread; in rice-eating countries, it is not and is eaten only as vegetable D. The rice-shortage problem in rice-eating countries like Asia can be solved by extensive white potato production. White potato, which is eaten only as vegetable in these countries is a good substitute for, or supplement to rice, as stressed by foreign and local specialists. An added advantage of white potato over rice is its greater versatility, with a per hectare yield four or five times more than rice. 3. According to the selection, which of the following is not true about white potato? A. It is grown extensively in bread-eating countries. B. It is eaten as a substitute for rice in rice-eating countries. C. It is more versatile than rice. D. It is a supplement to wheat bread in bread-eating countries. Selection 2 When someone speaks of Troy, he may be thinking of the city in one of three ways – as Troy of history, as Troy of literature, or as Troy of archaeology. Historically, we know that the ancient city of Troy, once called Ilium, was located in northwest Asia Manor, on the banks of the Hellespont. We don’t know the actual age of the city but relics discovered these have been proven to be as much as 5,000 years old. We know, too, that historical Troy was a rich and powerful city. It controlled all the commerce of the waters of the area and extended its power over much of Asia Minor. The Troy of literature, on the other hand, is the one first celebrated by the Greek poet Homer. His Odyssey tells the story of Ulysses, one of the heroes of the Trojan War, and his Iliad tells the complete story of the burning of Troy by the Greeks. Many later writers were inspired by Homer’s stories. They, in turn, wrote of the beauty of Helen of Troy, of the Greek “gift” to the Trojans of a giant wooden horse, and of the Greek warriors who won the war and rescued Helen. In archaeology, Troy is famous in still another way. Heinrich Schliemann’s excavations at Hisssarlik in 1891 uncovered the rich ruins of a burned city. He thought he had discovered the ruins of Homer’s Troy. In many later excavations, however, other archaeologists dug down through eight more cities buried below Schliemann’s. They decided that the city at the seventh level was actually the one of which Homer wrote. 4. Given below is an incomplete outline for the third paragraph of the selection. Choose the sub-topics which will best complete the outline. II. Troy of Literature A. Greek poet, Homer, first to celebrate Troy 1. Odyssey – story of Ulysses, hero of the Trojan War 2. Iliad – story of the burning of Troy B. ___________________________________________ 1. _________________________________________ 2. _________________________________________ 3. _________________________________________ A. B. The Trojan War C. B. Stories of Greek Warriors 1. Helen’s beauty 1. Homer’s influence on later writers 2. The Trojan’s gift 2. The beauty of Helen of Troy 3. Helen’s rescue 3. The Greeks gift to the Trojans B. B. Influence of Homer on Later Writers D. B. The Beauty of Troy 1. The beauty of Helen glorified 1. The beauty of Helen of Troy 2. The story of the giant wooden horse 2. The city’s wealth of power 3. Stories of Greed warriors 3. The courage of its warriors 5. On the whole, the selection is about A. ancient Troy C. the many ways by which Troy is known B. the great archeological excavations in Troy D. the rise and fall of Troy
  • 19. 19 6. Given below are sub-topics found in the selection. Choose the one which belongs to the major topic, Troy of Archeology. 1. Iliad, the story of Troy’s burning 2. Ilium, the ancient city of Troy 3. Heinrich Schliemann’s excavations 4. Odyssey, the story of Ulysses 5. Uncovering the eight more cities 6. The power of the city of Troy A. 1 and 4 B. 2 and 6 C. 3 and 5 D. 2 and 4 Selection 3 Dear Dr. Strauss: Under separate cover I am sending you a copy of my report entitled, “The Algernon-Gordon Effect: A Study of the Structure and Function of Increased Intelligence,” which I would like to have published. As you see, my experiments are completed. I have included in my report all of my formulae, as well as mathematical analysis in the appendix. Of course, these should be verified. Because of its importance to both you and Dr. Nemur (and need I say to myself, too?) I have checked and rechecked my results a dozen times in the hope of finding an error. I am sorry to say the results must stand. Yet for the sake of science, I am grateful for the little bit that I here add to the knowledge of the function of the human mind and of the laws governing the artificial increase of human intelligence. I recall your once saying to me that an experimental failure or the disproving of a theory was as important to the advancement of learning as a success would be. I know now that this is true. I am sorry, however, that my own contribution to the field must rest upon the ashes of the work of two men I regard so highly. Yours truly, Charles Gordon 7. What is Charles Gordon? A. lawyer B. scientist C. teacher D. mathematician 8. The clause “an experimental failure or the disproving of a theory was as important to the advancement of learning as a success would be” means that A. only success can advance learning B. failure slows down advancement of learning C. advancement of learning is important D. both experimental failure and success can advance learning 9. The phrase “the results must stand” as found in the third paragraph means that the results of the experiments A. must not be changed C. are wrong sometimes B. must keep changing D. need to be verified several times 10. The last paragraph suggests that his experiment has been A. disapproved B. successful C. a failure D. incomplete 11. What was Charles feeling while writing the letter? A. frustrated and sad C. relieved and thankful B. excited and happy D. nostalgic and resigned
  • 20. 20 Selection 4 The Man He Killed Had he and I but met By some old ancient inn, We would have sat us down to wet Right many a nipperkin! But ranged as infantry And staring face to face. I shot at him as he at me And killed him in his place. I shot him dead because – Because he was my foe, Just so: my foe of course he was; That’s clear enough; although He thought he’d ‘list, perhaps, Off-hand – like – just as I – Was out of work – had sold his traps – No other reason why. Yes, quaint and curious war is! You shoot a fellow down You’d treat, if met where any bar is, Or help to half-a-crown. 12. The person speaking in the poem is a A. bar tender B. murderer C. drunkard D. soldier 13. Which is the best summary of the poem? A. The two men met in the battlefield and became friends; they met later in an inn. B. After trying to kill each other in the battlefield, the two men in an inn; they became friends. C. The two men killed each other in the battlefield; they met in an inn before. D. The two men were enemies in the battlefield, so when they shot at one another, one of them got killed; they would have become friends had they met elsewhere. 14. The author must have written the poem to point out the A. danger of frequenting bars C. foolishness of war B. need to forgive one’s enemies D. hardship of being jobless 15. What is the best paraphrase of the last stanza? A. How tragic war is! It makes people not only kill their fellow men but also spend their time and money in bars. B. War is a strange thing! In the battlefield one kills a person whom he would treat as a friend elsewhere. C. Was should come to an end! It makes one kill another instead of befriending and helping him D. War is dreadful; it makes you kill even your own friends. Part III – GRAMMAR A. Complete each sentence by choosing the answer that will make it correct. Circle the letter of your answer. 1. He found the book I had lent him __________ on the table A. lain B. lay C. laid D. lying 2. Her father was one of the businessmen who __________ by the depression A. are ruined B. is ruined C. were ruined D. is going
  • 21. 21 3. Either French dressing or mayonnaise __________ well with tomatoes. A. goes B. go C. are going D. is going 4. Lea Salonga sings very well, doesn’t she? __________. A. Yes, she does. B. No, she does. C. Yes, she doesn’t D. No, she doesn’t 5. __________ every rule there is an exception. A. In B. To C. For D. Under 6. Anybody __________ it is to his advantage to have a college degree. A. know B. will know C. knows D. will 7. He hopes you __________ attend his graduation A. could B. would C. might D.will 8. I could wear my black or brown pair of shoes. Which do you think is __________? A. good B. better C. well D. best 9. The dog bit Linda __________ hard __________ she cried in pain. A. such – as B. too – that C. so – that D. such - that 10. When the battle was over, the victorious troops __________ their flag over the city. A. raised B. raises C. rise D. risen 11. The man was __________ conniving with the hold-up men. A. accused to B. accused for C. accused of D. accused by 12. “When is Mrs. Chavez planning to retire?” “Soon, I think. She __________ here for a long time. She’ll probably retire either next year or the year after that. A. worked B. had been working C. has been working D. is working 13. “Why did you buy all this sugar and chocolate?” “I __________ a cake for my friend’s birthday.” A. make B. am going to make C. will made D. will have made 14. Next week when there __________ a full moon, the ocean tides will be higher. A. is being B. is C. will D. has been 15. On July 20, 1969, Astronaut Neil Amstrong __________ down into the moon, the first person to set foot on another celestial body. A. was stepping B. stepped C. has stepped D. was step 16. Many years of intensive language study are required for non-native speakers to be able to qualify as interpreters. By the end of this year, Kim __________ English for three years, but he will still need more training and experience before he masters the language. A. will be studying C. will have been studying B. has studied D. has been studying 17. After ten unhappy years, Marissa finally quit the job. She __________ along with her boss for a long time before she finally decided to look for a new position. A. hadn’t been getting B. isn’t getting C. hasn’t been getting D. didn’t get 18. A minor earthquake occurred at 2:07 a.m. on January 3. Most of the people in the village __________ at that time and didn’t even know it had occurred until the next morning. A. slept B. had slept C. were sleeping D. sleep 19. Gina’s eyes burned and her shoulder ached. She __________ at the computer for five straight hours. Finally she took a break. A. is sitting B. has been sitting C. was sitting D. had been sitting
  • 22. 22 20. Inah and Jojie were mischievous children. They __________ tricks on their teachers which always got them into a lot of trouble. a. could play B. would play C. play D. played 21. You have to pay extra if you take too __________ with you. A. much luggage B. many luggages C. much luggages D. many luggage 22. At the news conference, several reporters didn’t get clear answers to __________ questions. A. theirs B. their C. his and hers D. his and her 23. Two-thirds of my __________ from the province. A. classmate is B. classmate are C. classmate is D. classmates are 24. If I could speak French, I __________ next year studying in France. A. would spend B. would have spent C. has spent D. will spend 25. If I __________ the same problems as you had as a child, I might not have succeeded in life as well as you have. A. have B. would have C. had D. should have 26. By measuring changing conditions in the atmosphere, __________ general weather patterns. A. meteorologists who are predicting C. predicting meteorologists B. meteorologists were predicted D. meteorologists can predict 27. Not until the end of the nineteenth century, __________ become a scientific discipline. A. plant breeding had B. did plant breeding C. plant breeding have D. have plant breeding 28. The cerebral cortex is __________ where the process of remembering faces takes place. A. the area is brained B. the area of the brain C. and a brain area D. brain area 29. By the end of the nineteenth century, Thomas Edison had invented the first practical light bulb, __________. A. a source of cheap electrical light C. a source light cheap electrically B. the light of electricity cheap source D. light with cheap electrically source 30. __________ about babies’ feelings is inferred from their expressions. A. What we know B. To be known by us C. Knowing D. Known 31. __________, communities are formed in a variety of ways. A. Created a division of labor C. Create a division of labor B. To create a division of labor D. Creation of division of labor 32. The condition necessary __________ this project have not been meet. A. for the complete B. of completion C. for completion of D. of complete 33. __________ are found in virtually every country in the world. A. Fruit flies and mosquitoes. C. When fruit flies and mosquitoes B. Now that fruit flies and mosquitoes D. Fruit flies and mosquitoes 34. Successful salespeople __________ and understand the needs of the market. A. products are thoroughly known C. thoroughly know their products are B. know their products thoroughly D. their products are thoroughly known 35. The number of members of the executive board in a big corporation is fixed by the bylaws __________ by the president. A. nevertheless B. despite C. instead D. not 36. Lasers __________ steel by focusing an intense beam on the metal. A. cutting B. cut C. to cut D. of cutting 37. __________ common nuclear reactions, cold fusion does not require a high temperature. A. Alike B. It is unlikely C. It is not like D. Unlike
  • 23. 23 38. Native American people arrived on the North American continent __________. A. since B. for C. before D. ahead 39. The ways of traveling __________ dramatically since the late nineteenth century. A. will have changed B. has changed C. have changed D. will change 40. How many of us __________ over complicated changes in the law! A. not frustrated C. have not become frustrated B. not become frustrated D. is not frustrated 41. A promissory note __________ anything without the trust deed. A. does mean B. means C. have not mean D. may not mean 42. Dairy farming is __________ leading agricultural activity in the United States. A. a B. at C. then D. none 43. Although thunder and lightning are produced at the same time, light waves travel faster, __________ so we see the lightning before we hear the thunder. A. than sound waves do C. do sound waves B. than sound waves are D. sound waves 44. Beef cattle __________ of all livestock for economic growth in certain geographic regions. A. the most are important C. the most important are B. are the most important D. that are the most important 45. The discovery of the half tone process in photography in 1881 made it __________ photographs in books and newspapers. A. the possible reproduction C. the most important are B. possible to reproduce D. that are the most important 46. Beginning in the Middle Ages, composers of Western music used a system of notating their compositions __________ be performed by musicians A. will B. that C. and when to D. so they could 47. Civil Rights are the freedoms and rights __________ as a member of a community, state or nation. A. may have a person C. a person may have B. may have a person who D. and a person may have 48. Richard Wright enjoyed success and influence __________ among Black American writers of his era. A. were unparalleled B. are unparalleled C. unparalleled D. the unparalleled 49. Franklin D. Roosevelt was __________ the great force of radio and the opportunity it provided for taking government policies directly to the people. A. as the first President he understood fully C. the first President to fully understand B. the President fully understand D. the first President to understand fully 50. During the late fifteenth century, __________ of the native societies of America had professions in the fields of arts and crafts. A. only a few B. a few but C. few but only D. a few only B. Write the number that corresponds to the word or phrase which makes the sentence incorrect. Write number 5 if the sentence has no error. _____ 1. You are the very-person who I saw. 1 2 3 4 _____ 2. She spoke clear and made a good impression on the audience. 1 2 3 4 _____ 3. We do our shopping in Makati because of their big stores there. 1 2 3 4 _ ____ 4. I think a hundred pesos are too much for that toy. 1 2 3 4
  • 24. 24 _____ 5. Exploding firecrackers during the New Year are a time-honored tradition in our country. 1 2 3 4 _____ 6. If I was you I would think twice before resigning. 1 2 3 4 _____ 7. If he had only knew that the U.S. would invade Panama he would have been on the alert. 1 2 3 4 _____ 8. Since its establishment in 1982 the company is very progressive. 1 2 3 4 _____ 9. Ben, together with Mon and Ronnie have been chosen to play for the national team. 1 2 3 4 _____ 10. Rizza’s mother scolded her when she break the imported vase. 1 2 3 4 _____ 11. My teacher and friend, Mr. Torres are with my groupmates 1 2 3 4 _____ 12. The security guard assured us that everything will be taken cared of. 1 2 3 4 _____ 13. We should not allow aliens to exploit our natural resources. 1 2 3 4 _____ 14. One of the puppies looks different from it’s parents. 1 2 3 4 _____ 15. “You shouldn’t meddled in the controversy,” Aunt Sophia said. 1 2 3 4 _____ 16. Each student who attends the orientation last Saturday was requested to fill out a personal data sheet. 1 2 3 4 _____ 17. When the policeman saw the robber, he handcuffed the man who wished he has not run that direction. 1 2 3 4 _____ 18. Although Dan had known the combination to the lock, he never told anyone 1 2 3 4 _____ 19. Please notify we committee members of the next meeting. 1 2 3 4 _____ 20. Ask them or us anything you want to know you want to know about the computer. 1 2 3 4 _____ 21. She is usually complementary to whomever she is with. 1 2 3 4 _____ 22. Either of the girls might have a chance to sing their song on television 1 2 3 4 _____ 23.Every man, woman, and child in the Philippines are guaranteed certain rights. 1 2 3 4 _____ 24. I don’t know which is worst, packing to go away or unpacking. 1 2 3 4 _____ 25. I’d advise you to follow your doctor’s advise. 1 2 3 4 _____ 26. In some species of fish, such the three-spined stickleback, the male, not the female, performs 1 2 3 the task of caring for the young. 4 _____27. When she retires in September 1989, tennis champion Christine Evert was the most famous 1 2 3 woman athlete in the United States. 4 _____ 28. The ancient Romans used vessels equipped with sails and banks of oars to transporting their 1 2 3 4 armies. _____29 Dinosaurs are traditionally classified as cold-blooded reptiles, but recent evidence based on 1 2 eating habits, posture, and skeletal structural suggests some may have been warm-blooded. 3 4
  • 25. 25 _____30 In human beings, as in other mammal hairs around the eyes and ears and in the nose, prevent 1 2 3 dust, insects, and other matter from entering these organs. 4 _____31. The works of the author Herman Melville are literary creations of a high order, blending fact, 1 2 3 fiction, adventure, and subtle symbolic. 4 _____32. Each chemical element is characterized to the number of protons that an atom of that element 1 2 3 contains, called its, atomic number. .4 _____33. The body structure that develop in birds over millions of well designed for flight, being both 1 2 3 lightly in weight and remarkably strong. 4 _____34. From 1905 to 1920, American novelist Edith Wharton was at the height of her writing career, 1 2 3 publishing of her three most famous novels. 4 _____ 35. In the early 20th century, there was considerable interesting among sociologists in the fact that 1 2 in the United States the family was losing its traditional roles. 3 4 _____36. Although pure diamond is colorless and transparent, when contaminated with other material it 1 2 3 may appear in various color ranging from pastels to opaque black. 4 _____37. Comparative anatomy is concerned to the structural differences among animal forms. 1 2 3 4 _____38. A seismograph records oscillation of the ground caused by seismic waves, vibrations that travel 1 2 from its point of origin through the Earth or along its surface. 3 4 _____39. Electric lamps came into widespread use during the early 1900 and have replaced other type of 1 2 fat, gas, or oil lamps for almost every purpose. 3 4 _____40. Emily Dickinson, among the greatest woman poets in the English language, died with all of her 1 2 3 poems unpublished, except for seven that appeared in the publications of limited circulation. 4 _____41. Protecting coral reefs is difficult because some of the corals are very fragile; even the touch of 1 2 3 4 the diver’s hand can kill it. _____42. To improvise effectively, a musician must thorough understand the conventions of a given 1 2 3 4 musical style. _____43. Leaves are believed to be one of the best substance to form compost piles. 1 2 3 4 _____44. Wood is an excellent resource for heating homes cooking food, and build houses. 1 2 3 4 _____45. River water pollution is often indicator by algae distribution. 1 2 3 4 _____46. Modern farms are much larger than that of former times. 1 2 3 4 _____47. It is often not until the end of their lives that famous people receive the recognize they deserve. 1 2 3 4 _____48. Usually the climate in mountainous areas becomes much windy at higher altitudes. 1 2 3 4
  • 26. 26 _____49. A physical chemist is required to function simultaneously as a physicist, chemist and 1 2 3 4 mathematician. _____50. A baby gorilla is a shy, friendly animal that like attention. 1 2 3 4 THE TRADITIONAL CONCERNS: WORD CHOICE AND GRAMMAR A. Sentence Fragment. A sentence expresses a logically complete idea. If the idea is not complete – if your reader is left wondering what you mean – you probably have omitted some essential element. ? She spent her first week on the job as a researcher. Compiling information from digests and journals. ? Because the operator was careless. The new computer was damaged. B. Comma Splice. Two complete ideas (independent clauses), which should be separated by a period or a semicolon, are incorrectly joined by a comma. ? Sarah did a great job, she was promoted. ? This is a new technique, therefore, some people mistrust it. C. Run-on Sentence. A sentence that crams, too many ideas without needed breaks or pauses. ? The hourglass is more accurate than the water clock because water in a water clock must always be at the same temperature to flow at the same speed since water evaporates and must be replenished at regular intervals, thus being less effective than the hourglass for measuring time. D. Faulty Coordination. Ideas of equal importance are joined, within simple or compound sentences, with coordinating conjunctions: and, but, or, nor, for, so, and yet. ? Joseph had a drinking problem and he dropped out of school. ? I will try and help you. ? The climax in jogging comes after a few kilometers and I can no longer feel stride after stride and it seems as if I am floating and jogging becomes almost a reflex and my arms and legs continue to move and my mind no longer has to control their actions. E. Faulty Subordination. Proper subordination shows that a less important idea is dependent on a more important idea using subordinating conjunctions: because, so, if, unless, after, until, since, while, as and although. ? Television viewers can relate to an athlete they idolize and they feel obliged to but the product endorsed by their hero. ? This teacher is often late for work, and he writes illogical reports, and he is a poor manager, and he should be fired. ? This job which I took when I graduated from college, while I waited for a better one to come along, which is boring, where I’ve gained no useful experience, make me anxious to quit. F. Faulty Agreement – Subject and Verb. The subject should agree in number with the verb. In more complicated sentences – those in which the subject is separated from its verb by other words – we sometimes lose track of the subject-verb relationship. ? The lion’s share of diesels are sold in Europe. ? A system of line extend horizontally to form a grid. ? Each of the crew members were injured. ? Everyone in the group have practiced long hours. ? The committee meet weekly to discuss ness business. G. Faulty Agreement – Pronoun and Referent. A pronoun can make sense only if it refers to a specific noun (its referent or antecedent), with which it must agree in gender and number. ? Anyone can get their degree form that college. ? Each candidate described her plans in details.
  • 27. 27 H. Faulty or Vague Pronoun Reference. Whenever a pronoun is used, it must refer to one clearly identified referent; otherwise the message will be confusing. ? Rolly told Ramon that his wife loves him. ? He drove away from his menial job, boring lifestyle, and damp apartment, happy to be leaving it behind. ? The problem with our defective machinery is compounded by the operator’s incompetence. That annoys me! I. Faulty Pronoun Case. A pronoun’s case (nominative, objective, or possessive) is determined by its role in a sentence: as subject, object, or indicator of possession. ? Whom is responsible to who? ? The debate was between Marina and I. ? Us teacher are accountable for our decisions. ? A group of we teachers will fly to the convention. J. Faulty Modification. A sentence’s word order (syntax) helps determine its effectiveness and meaning. ? Dialing the phone, the cat ran out the open door. ? While walking, a cold chill ran through my body. ? Gina typed another memo on our computer that was useless. ? She volunteered immediately to help the landslide victims in Leyte. K. Faulty Parallelism. To reflect relationships among items of equal importance, express them in identical grammatical form. ? We here highly resolve … that government of the people, which the people created and maintain, serving the people shall not perish from the earth. ? The new teacher is enthusiastic, skilled, and you can depend on her. ? In her new assignment, she felt lonely and without a friend. ? She sleeps well, jogs daily, as well as eating high-protein foods. L. Sentence Shifts. Shifts in point of view damage coherence. ? When you finish the jog, one will have a sense of pride. ? One should sift the flour before they make the pie. ? He delivered the plans for the apartment complex, and the building site was also inspected by him. ? She delivered the blueprints, inspected the foundation, wrote her report, and takes the afternoon off. ? Unscrew the valve and then steel wool should be used to clean the fitting. ? Jim wonders, if he will get the job and will he like it? M. Effective Punctuation. Punctuation marks are like road signs and traffic signals that give a simple way of making one be understood. Compare the two letters below. Dear Jack, I want a man who knows what love is all about. You are generous, kind, and thoughtful. People who are not like you admit to being useless and inferior. You have ruined me for other men. I yearn for you. I have no feelings whatsoever when we’re apart. I can be forever happy – will you let me be yours? Jill Dear Jack, I want a man who knows what love is. All about you are generous, kind, and thoughtful people who are not like you. Admit to being useless and inferior. You have ruined me. For other men I yearn! For you. I have no feelings whatsoever. When we’re apart I can be forever happy. Will you let me be? Yours, Jill
  • 28. 28 Practice Exercises A. The following letter to the editor of a newspaper has twelve errors in subject-verb agreement and pronoun-antecedent agreement. Find and correct the errors. Use forms that are correct for formal English. To the Editor, The Times Many parts of our once-beautiful city is starting to look like mini garbage dumps. You will recall that legislation requiring recycling within the city limits were passed last year, and the mayor and other local politicians encourages us to recycle, but in my apartment complex there is no bins for recycling. The result is that people take no responsibility for his own actions, and everyone tosses their trash and recyclables (glass, plastic bottles, cans, etc) right in with the food that is being thrown away. Neither the manager of the complex nor the owners of the building has bought any new containers for the items that are supposed to be recycled. So what else can everyone do but mix their trash together? Either the manager or the owners is responsible for breaking the law here. Not us! Meanwhile, trashcans in the downtown area is overflowing with garbage, and vacant lots all around the city is littered with soda cans, broken glass, and paper. The owner and publisher of your newspaper, Mr. Romeo Gomez, have always been a supporter of clean environment. I urge your paper to take leadership in solving this problem. B. Read the following paragraph about the Judge Crater mystery. Correct three errors I parallelism with active or passive voice On the evening of August 6, 1930, Judge Force Crater, a wealthy, successful, and good-looking New York lawyer disappeared without a trace. Earlier in the evening he had been seen with friends at a Manhattan restaurant. At 9:10 P.M. he left the restaurant, hailed a taxi, and was driven away. No one ever saw or heard from him again. It was ten days before he was even reported missing. On August 16, his wife called his courthouse, the secretary was asked about his whereabouts, and learned that he was probably off on political business. This news reassured Mrs. Crated somewhat, but when he still hadn’t turned up by August 26, a group of his fellow judges started an investigation. A grand jury was convened, but its members could not come to any conclusion as to what had happened to Judge Crater. They theorized that the judge might have developed amnesia, run away voluntarily, or been a crime victim. He wife disagreed with the first two possibilities, holding that he had been murdered by someone in the Tammany Hall organization, the political machine that controlled New York City at the that time. The mystery remains unsolved to this day. He could have been killed by a Tammy hall hiree, a girlfriend could have murdered him, or kidnapped by an organized crime group. He might in fact have suffered from amnesia, or he might have planned his own disappearance. Reports of Judge Crater sightings have continued to surface over the last sixty years. Words and Expressions Commonly Misused. Many words and expressions are not so much bad English as bad style, the commonplaces of careless writing.  All right. Idiomatic in familiar speech as a detached phrase in the sense “Agreed,” or “O.K.”  Alternate, Alternative. The words are not always interchangeable as nouns or adjectives. The first means every other one in a series; the second, one of two possiblilities.  Among, Between. When more than two things or persons are involved, among is usually called for. When, however, more that two are involved but each is considered individually, between is preferred.  Anticipate. Use expect in the sense of simple expectation  Anybody. In the sense of “any person,” not to be written as two words, Any body means “any corpse.”  Anyone. In the sense of “anybody,” written as one word. Any one means “any single person.”  As to whether. Whether is sufficient.  As yet. Yet nearly always is as good, if not better.  Careless. The dismissive “I couldn’t care less” is often used with the shortened “not” mistakenly omitted: “ I could care less.”
  • 29. 29  Certainly. Used indiscriminately by some speakers much as other use very, in an attempt to intensify any and every statement.  Consider. Not followed by as when it means “believe to be.”  Contact. As a transitive verb, the word is vague and self-important. Do not contact people; get in touch with them, look them up, phone them, find them, or meet them.  Cope. An intransitive verb used with with. In formal writing, one doesn’t “cope,” one “copes with” something or somebody.  Disinterested. Means “impartial.” Do not confuse it with uninterested which means “not interested in.”  Each and every one. Avoid, except in dialogue.  Effect. As a noun, means “result”; as a verb, means “to bring about,” “to accomplish” (not to be confused with affect, which means “to influence”)  Enthuse. An annoying verb growing out of the noun enthusiasm. Not recommended.  Factor. A hackneyed word; the expressions of which is a part can usually be replaced by something more direct and idiomatic  Farther, Further. Farther serves best as distance word, further as a time or quantity word  Finalize. A pompous, ambiguous verb.  Hopefully. This once-useful adverb meaning “with hope” has been distorted and is now widely used to mean “I hope” or “it is to be hoped.”  However. Avoid starting a sentence with however when the meaning is “nevertheless.” The word usually serves better when not in first position.  Imply, Infer. Not interchangeable. Something implied is something suggested or indicated, though not expresses. Something inferred is something deduced from evidence at hand.  In regard to. Often wrongly written in regards to. But as regards is correct, and means the same thing.  Insightful. The word is suspicious overstatement for “perceptive.”  Irregardless. Should be regardless.  -ize. Do not coin verbs by adding this tempting suffix. Many good and useful verbs do end in –ize: summarize, harmonize, fertilize. But there is a growing list of abominations: prioritize, utilize, personalize, finalize.  Kind of. Except in familiar style, not to be used as a substitute for rather or something like.  Like. Not to be used for the conjunction as. Like governs nouns and pronouns; before phrases and clauses the equivalent word is as.  Nice. A shaggy, all-purpose word, to be used sparingly in formal composition.  Ongoing. A mix of “continuing” and “active” and is usually superfluous.  One of the most. There is nothing wrong with the grammar; the formula is simply threadbare.  Prestigious. Often an adjective of last resort. It’s in the dictionary, but that doesn’t mean you have to use it.  Respective, Respectively. These words may usually be omitted with advantage.  Secondly, thirdly, etc. Unless you are prepared to begin with firstly, and defend it (which will be difficult), do not prettify numbers with –ly.  Thanking you in advance. This sounds as if the writer meant, “It will not be worth my while to write to you again.”  The foreseeable future. A cliché, and a fuzzy one.  The truth is … The fact is … A bad beginning for a sentence. If you feel you are possessed of the truth, or of the fact, simple state it. Do not give advance billing.  Very. Use this word sparingly. Where emphasis is necessary, use words strong in themselves.
  • 30. 30 C. Correct the misused words and expressions in the sentences below. 1. An agreement was reached among the two companies to work on the same project as equal partners. 2. The schedule has not been finalized yet. 3. The office managers considered the new policy as rigid and unreasonable. 4. The case of the dismissed employee should be a lesson to each and everyone of us. 5. After considering the applicant’s qualifications, I can say that he is a man who is knowledgeable and well-trained for the job. 6. The project is going to be finished as scheduled. More importantly, the company is able to save one-third of the original budget. 7. Irregardless of the problems encountered at the project site, we implemented the plan as scheduled. 8. This is a nice proposal to improve the flow of communications within the company. 9. The office cannot authorize nor justify overtime work rendered on Sundays. 10. Our company needs to prioritize the requests made by each department regarding office supplies. 11. I would like to assure you that the project will be completed as scheduled. Personally, I believe that all the problems have been addressed adequately. 12. The project director stated that no new projects will be approved until the end of the year. 13. The truth is the workers are overworked and underpaid. 14. The proposal is so interesting that the company agrees to meet with the people concerned. 15. Most all the time, the reports of the engineers do not include the day-to-day progress of work in the job site. D. Filipinism is language of the Philippine English variety. As such, they may be understood by Filipino speakers but considered unidiomatic by native English speakers. How do you make the following Filipinisms more idiomatic? 1. I drink medicine daily. 2. He declared his love for her. 3. We called a police. 4. Will you pass first? 5. I am vacant. 6. I dress up before breakfast. 7. My watch is advanced/behind. 8. The maid will serve us. 9. Come already, we shall be late. 10. The student’s pension is enough for his needs. 11. I arrange my bed every morning. 12. Maria has a bad custom of copying. 13. Open the light. 14. Repeat the sentence again. 15. You are becoming fat. 16. She is my co-teacher. 17. How is your Mrs.? 18. I am finished. 19. I’ll tell you to my mother. 20. How is life getting along with you?
  • 31. 31 READING IN THE CONTENT AREAS I. Read each excerpt and choose the best answer to each question that follows. American History (An excerpt, by Judith Ortiz-Cofer, 1992) 5 10 15 The day Mr. DePalma came out into the cold and asked us to line up in front of him was the day that President Kennedy was shot. Mr. DePalma, a short, muscular man with slicked-down black hair, was the science teacher, P.E. coach, and disciplinarian at P.S. 13. He was the teacher to whose homeroom you got assigned if you were a troublemaker, and the man called out to break up playground fights, and to escort violently angry teenagers to the office. And Mr. DePalma was the man who called your parents in for a “conference.” That day, he stood in front of two rows of mostly black and Puerto Rican kids, brittle from their efforts to “keep moving” on November day that was turning bitter cold. Mr. DePalma, to our complete shock, was crying. Not just silent adult tears, but really sobbing. There were a few titters from the back of the line where I stood shivering. “Listen,” Mr. DePalma raised his arms over his head as if he were about to conduct an orchestra. His voice broke, and he covered his face with his hands. His barrel chest was heaving. Someone giggled behind me. “Listen,” he repeated, “something awful has happened.” A strange gurgling came from his throat, and he turned around and spat on the cement behind him. “Gross,” someone said, and there was a lot of laughter. “The President is dead, you idiots. I should have known that wouldn’t mean anything to a bunch of losers like you kids. Go home.” *********** 1. Where does the action of the story take place? A. in the gym of a community center C. on the front steps of a federal courtroom B. on the playground of a public school D. in the yard of a federal prison 2. Mr. DePalma might best be described as A. softhearted B. funny C.conceited D. strict 3. Mr. DePalma cried because he was A. hurt that people were making fun of him B. upset about the death of the president C. sick to his stomach and unable to breathe D. frightened that war was about to be declared 4. How does Mr. DePalma feel about the speaker and her friends? A. indifferent B. proud C. affectionate D. disgusted 5. The author develops the character of Mr. DePalma in all of the following ways except A. revealing what Mr. DePalma says to other characters B. disclosing what Mr. DePalma secretly thinks C. showing how Mr. DePalma interacts with others D. describing what Mr. DePalma looks like
  • 32. 32 Suburban Prophecy By Howard Nemerov, 1977 5 10 On Saturday, the power-mowers whine Begins the morning. Over this neighborhood Rises the keening, petulant voice, begin Green oily teeth to chatter and munch the cud. Monsters, crawling the carpets of the world, Still send from underground against your blades The roots of things battalions green and curled And tender, that will match your blades with blades Till the revolted throats shall strangle on The tickle of their dead, till straws shall break Crankshafts like camels, and the sun go down On dinosaurs in swamps. A night attack Follows and by the time the Sabbath dawns All armored beasts are eaten by their lawns. ********** 6. In the first stanza, the poet uses words such as whine, voice, teeth, chatter, and munch to suggest that the power-mowers are A. alive B. in need of repair C. like cows D. very powerful 7. The imagery in the first stanza most appeals to the reader’s sense of A. sight B. touch C. hearing D. taste 8. The phrase “your blades” in line 8 refers to A. grass B. gardeners C. lawnmowers D. monsters 9. The action of the poem takes place in the time period of A. an afternoon B. a morning C. twenty-four hours D. a week 10. The events and imagery of the second stanza create an atmosphere in which A. suburbanites use their lawnmowers against each other B. the people pushing the mowers are completely exhausted C. the smell of freshly cut grass lingers in the air D. lawns and lawnmowers are battling 11. Throughout the poem, Nemerov creates an interesting image of lawnmowers by comparing them to A. animals B. other machines C. people D. plants 12. The poet is ridiculing A. suburbanites who turn into monsters C. lawnmowers and other modern machines B. suburbanites who keep perfect lawns D. realtors who sell house without lawns 13. In one description, Nemerov makes a comparison using simile and alliteration. Which of the following descriptions represents both of these techniques? A. “crawling the carpets of the world” B. “Green oily teeth to chatter and much the cud” C. “Crankshafts like camels” D. “and the sun go down/ on dinosaurs in swamps”
  • 33. 33 “Beauty Marks” Review of Smile Music by Marvin Hamilsch Book and Lyrics by Howard Ashman by William A. Henry III, Time, December 8, 1986 5 10 15 This season’s first four new American musicals all closed the seek they opened, continuing a daunting three-year run of almost unrelieved financial failure for what used to be Broadway’s mainstay. Staged with varying degrees of artistry, the ill-fated shows shared one disabling presumption: musicals must be “about” something beyond melody and romance … All suffocated under the weight of their ambition. One might expect the same fate to befall Smile, adapted from a 1975 Michael Ritchie film that satirized beauty pageants. The narrative, centering on girls who are strangers, inevitably lacks complex relationships and love interest. Moreover, it is difficult to write a parody much funnier than the real Miss America proceedings. And it is hard to keep audiences interested in the climax – which entrant will win – after repeatedly telling them it shouldn’t matter. Curiously, Smile works. It is a swift-paced, skillfully performed and thoroughly professional entertainment that balances amusement at the shallow ambitions of the characters with respect for the depth of their feelings. Composer Marvin Hamlisch (A Chorus Line) and Author-Lyricist Howard Ashman (Little shop of Horrors) have written touching songs for the stars, Anne Marie Bobby as a sweet, awkward A student who realizes she is out of her element at the pageant and Jodi Benson as a wanderer who is prematurely wise in the ways of selling herself, including a talent-show “dramatic reading” that turns into a strip-tease. Smile may not be a landmark, but it is a pleasure. ********** 14. According to the critic, musicals were, at the time, A. as popular as ever C. unsuccessful on Broadway B. gaining popularity each year D. not as popular as films 15. Lines 5 state that “All suffocated under the weight of their ambition.” The critic includes this descriptive language to suggest that A. the directors and actors were not experienced B. the musical relied too much on light humor C. producers were too concerned with winning awards D. the musicals tried to make a statement as well as entertain 16. The critic states that the musical “balances amusement at the shallow ambitions of the characters with respect for the depth of their feelings” (lines 12-13). This expresses a contrast between the audience’s A. feelings about Miss America and patriotism B. depth of understanding of the two main characters C. mixed emotions about the characters’ struggles D. favorable opinion of Smile 17. The critic believes the music is A. too humorous for the theme C. appropriate for the characters B. shallow and without purpose D. outstanding and destined to be popular 18. Which of the following statements describes the critic’s opinion of Smile? A. Although Smile is not a great musical, it is enjoyable. B. Smile is an example of why musicals are no longer popular. C. Smile unjustly criticizes the Miss America pageant. D. Since Smile is about women, it will surely be popular. 19. According to the experience of Broadway sows for three years running, the type of production that would have been most likely to fail would be a A. comedy about marriage C. one-woman concert B. drama about Nazi Germany D. musical about a person’s personal triumph 20. The statement “Curiously, Smile works” reveals the critic’s A. total disgust C. lack of faith in the audience B. dislike of the musical D. surprise at the musical’s success